Anda di halaman 1dari 76

Copyright 2014 Delhi Academy of Medical Sciences, All Rights Reserved.

1/76
Test Information
Test Name MDMS SWTS-PHYSIOLOGY Total Questions 200

Test Type Examination Difficulty Level Difficult

Total Marks 600 Duration 120minutes

Test Question Language:- ENGLISH

(1). Planning and programming of movements is done by which part of brain?

a. Vestibulocerebellum

b. Pre motor cortex

c. Spinocerebellum

d. Basal ganglia

Solution. Ans-1: (d) Basal ganglia Ref.: Read the text below Sol :
Planning and programming of movements are done by both cerebrocerebellum and basal ganglia.
Studies have made it clear that neurons in the basal ganglia, like those in the lateral portions of the cerebellar hemispheres, discharge
before movements begin.
These observations, plus careful analysis of the effects of diseases of the basal ganglion in humans and the effects of drugs that destroy
dopaminergic neurons in animals, have led to the concept that the basal ganglia are involved in the planning and programming of
movement or, more broadly, in the processes by which an abstract thought is converted into voluntary action.

Correct Answer. d

(2). Duration of second heart sound is?

a. 0.12 sec

b. 0.1 sec

c. 0.15 Sec

d. 02. Sec

Solution. Ans-2: (a) 0.12 sec Ref.: Read the text below Sol :

Correct Answer. a

Copyright 2014 Delhi Academy of Medical Sciences, All Rights Reserved. 2/76
(3). The term Mileu interior was given by?

a. Knut schmidt-nielsen

b. George Bartholomew

c. Claude Bernard

d. Walter Cannon

Solution. Ans-3: (c) Claude Bernard Ref.: Read the text below Sol :
Milieu intrieur or interior milieu, from the French, milieu intrieur (the environment within), is a phrase coined by Claude Bernard to
refer to the extra-cellular fluid environment, and its physiological capacity to ensure protective

Correct Answer. c

(4). Which non proteinous substance has highest normal blood level?

a. Urea

b. Creatinine

c. Uric acid

d. Ammonia

Solution. Ans-4: (a) Urea Ref.: Read the text below Sol :
Substance Normal Level
Urea 20-40 mg/dl

Creatinine 05.-0.9 mg/dL (female), 0.6-1.2 mg/dL (male)

Uric acid 2.5-5.6 mg/dL (female), 3.1 7.0 mg/dL (male)

Ammonia 19-60 ug/dL

Correct Answer. a

(5). Dietary fat, after being processed, is extruded from the mucosal cells of the gastrointestinal tract into the lymphatic ducts in the form of

a. Monoglycerides

b. Diglycerides

c. Triglycerides

d. Chylomicrons

Solution. Ans-5: (d) Chylomicrons Ref: Read the text below Sol :
Long-chain fatty acids are extruded from enterocytes in the form of chylomicrons into the lymphatic system.
Triglycerides are hydrolyzed to monoglycerides and taken into mucosal cells. If the fatty acids are short chains (less than 10 to 12 carbon
atoms), they are extruded in the form of free fatty acids into the portal blood.
Chylomicrons represent triglycerides and esters of cholesterol that have been invested in the intestinal mucosa
with a coating of phospholipid, protein, and cholesterol.

Correct Answer. d

Copyright 2014 Delhi Academy of Medical Sciences, All Rights Reserved. 3/76
(6). Contraction of the gallbladder is correctly described by which of the following statements?

a. It is inhibited by a fat-rich meal

b. It is inhibited by the presence of amino acids in the duodenum

c. It is stimulated by atropine

d. It occurs in response to cholecystokinin

Solution. Ans-6: (d) It occurs in response to cholecystokinin


Ref: Read the text below
Sol :
Cholecystokinin is released from the upper small intestine in response to partially hydrolyzed dietary lipids and
proteins and promotes gallbladder emptying. Gallbladder contraction and sphincter of Oddi relaxation are
necessary for delivery of bile into the duodenum.
These muscular actions are under both hormonal and neural control.
Cholecystokinin contracts gallbladder smooth muscle by a direct action on the muscle and through activation
of vagal afferent fibers leading to a vago-vagal reflex.
Relaxation of sphincter of Oddi smooth muscle occurs via activation of inhibitory enteric nerves. Vagal
stimulation, which is cholinergically mediated and blocked by atropine, also promotes gallbladder contraction.

Correct Answer. d

(7). The greatest percentage of blood volume is found in the

a. Heart

b. Aorta

c. Capillaries

d. Venules and veins

Solution. Ans-7: (d) Venules and veins


Ref: Read the text below
Sol :
The total circulating blood volume is approximately 70 mL/kg, about two-thirds of which is found in the
systemic veins and venules.
A significant volume of blood (15%) is found in the pulmonary circulation.
The large volume of blood found on the venous side of the circulation is used to adjust circulating blood
volume.
For example during hemorrhage, contraction of the veins and venules of the skin increases the amount of
blood available for perfusion of the heart and brain.

Correct Answer. d

(8). The diastolic pressure equals

a. 0 mmHg

b. 5 mmHg

c. 70 mmHg

d. 90 mmHg

Solution. Ans-8: (c) 70 mmHg


Ref: Read the text below
Sol :
Diastolic pressure is the lowest pressure observed in the aorta during each cardiac cycle.
It occurs at the end of isovolumic contraction, just before ejection of blood from the left ventricle begins.
In this case, isovolumic contraction ends when the pressure within the ventricle rises to 70 mmHg.
The term diastolic pressure always refers to arterial blood pressure. In the diagram, the left ventricular enddiastolic
pressure equals 5 mmHg.

Correct Answer. c

Copyright 2014 Delhi Academy of Medical Sciences, All Rights Reserved. 4/76
(9). Closure of the aortic valve occurs at the onset of which phase of the cardiac cycle?

a. Isovolumetric contraction

b. Rapid ejection

c. Protodiastole

d. Isovolumetric relaxation

Solution. Ans-9: (d) Isovolumetric relaxation


Ref: Read the text below
Sol :
Closure of the semilunar valves (aortic and pulmonic valves) marks the beginning of the isovolumetric
relaxation phase of the cardiac cycle.
During this brief period (approximately 0.06 s), the ventricles are closed and myocardial relaxation, which
began during protodiastole, continues. Intraventricular pressure falls rapidly, although ventricular volume
changes little.
When intraventricular pressure falls below atrial pressure, the mitral and tricuspid valves open and rapid
filling of the ventricles begins.

Correct Answer. d

(10). Which of the following statements about the vasomotor centre (VMC) is true

a. Independent of corticohypothalamic inputs

b. Influenced by baroreceptor signals but not by chemoreceptors

c. Acts alongwith the cardiovagal center to maintain blood pressure

d. Essentially silent in sleep

Solution. Ans-10: (c) Acts alongwith the cardiovagal center to maintain blood pressure
Ref: Read the text below
Sol :
The vasomotor centre (VMC) or the rostral ventrolateral medulla (RVLM) is one of the major excitatory input to the
sympathetic nerves controlling the vasculature and the heart. The RVLM is influenced by a number of factors:-
Direct stimulation
CO2
Hypoxia
Excitatory inputs
Cortex via the hypothalamus
PAG (periacqueductal gray in the mid brain)
Brain stem reticular formation
Pain pathways
Somatic afferents (somatosymapathetic reflex)
Carotid and aortic chemoreceptors
Inhibitory inputs
Cortex via the hypothalamus
Caudal ventrolateral medulla (CVLM)
Raphe nucleus (medulla)
Lung inflation afferents (inflation of the lungs causes vasodilatation and a
decrease in blood pressure- this response is mediated via vagal afferents from the
lungs that inhibit vasomotor discharge)
Carotid, aortic and cardiopulmonary baroreceptors (acting via the CVLM)
In view of the above, the VMC is definitely not independent of corticohypoyhalamic inputs. The VMC is influenced by both the
chemoreceptors (which act directly on the RVLM) and the baroreceptors (which act via the CVLM, inhibiting the RVLM). The VMC has a
close association with cardiac vagal motor neurons in the nucleus ambiguus, also in the medulla. The VMC continues to function in sleep.

Correct Answer. c

Copyright 2014 Delhi Academy of Medical Sciences, All Rights Reserved. 5/76
(11). Tears are lost at which of the following dehydration percentage:

a. 5%

b. 7%

c. 10%

d. 12%

Solution. Ans-11: (b) 7% Ref: Read the text below Sol :


Although tears are lost at 7% dehydration, when calculating fluid deficit needs it is recommended to round up to 10%.

Correct Answer. b

(12). In hyponatremic states, total body water is

a. Increased

b. Normal

c. Reduced

d. All of the above are possible

Solution. Ans-12: (d) All of the above are possible Ref: Read the text below Sol :
Low body sodium can occur as a primary sodium loss or as a sodium and water loss.
If more water than sodium is lost, a hypertonic state will occur.
If more sodium than water is lost, a hypotonic state will occur. If the same proportion of water and sodium is lost, normal osmotic
tension will remain.

Correct Answer. d

(13). Plasma volume is best evaluated by using?

a. Evan blue

b. Inulin

c. Mannitol

d. Radiolabelled water

Solution. Ans-13: (a) Evan blue Ref.: Read the text below Sol :
T-1824 or Evans Blue, often incorrectly rendered as Evan's Blue, is an azo dye which has a very high affinity for serum albumin. Because
of this, it can be useful in physiology in estimating the proportion of body water contained in blood plasma

Correct Answer. a

Copyright 2014 Delhi Academy of Medical Sciences, All Rights Reserved. 6/76
(14). Golgi tendons and muscle fibre ratio is?

a. 1:7

b. 1:13

c. 1:25

d. 1:40

Solution. Ans-14: (b) 1:13 Ref.: Read the text below Sol :
This organ consists of a net-like collection of knobby nerve endings among the fascicles of a tendon. There are 3-25 muscle fibers per
tendon organ.
The fibers from the Golgi tendon organs make up the Ib group of myelinated, rapidly conducting sensory nerve fibers.

Correct Answer. b

(15). Blood pressure changes by ____ above or below heart level?

a. 0.55 mm of Hg/cm

b. 0.75 mm of Hg/cm

c. 0.76 mm of Hg/cm

d. 0.77 mm of Hg/m

Solution. Ans-15: (d) 0.77 mm of Hg/m Ref.: Read the text below Sol :
The blood pressure increases or decreases by 0.77 mm of Hg/cm for every centimeter below or above the level of heart.

Correct Answer. d

(16). Hematocrit in veins is higher than arteries because?

a. Increased number of particles

b. Slow flow

c. Lymphatic absorption

d. Larger amount of blood

Solution. Ans-16: (a) Increased number of particles Ref.: Read the text below Sol :
Veins have higher hematocrit compared to arteries because due to addition of chloride ions, bicarbonate ions and CO2 which makes
blood osmolarity in veins to increase and therefore red cells take in water and hematocrit in veins increase.
This is major reason, minor reason is absorption of some fluid as lymphatics.

Correct Answer. a

Copyright 2014 Delhi Academy of Medical Sciences, All Rights Reserved. 7/76
(17). Action of pineal gland is?

a. Circadian rhythm

b. Secretion of TSH

c. Control of the impulses

d. Central control of autonomic nervous system.

Solution. Ans-17: (a) Circadian rhythm Ref.: Read the text below Sol :
Some pineal indoleamines, including melatonin and enzymes for their biosynthesis (e.g. serotonin N-acetyltransferase) show circadian
rhythms in concentration.
The level rises during darkness, and falls during the day, when secretion may be inhibited by sympathetic activity. It is thought that the
intrinsic rhythmicity of an endogenous circadian oscillator in the suprachiasmatic nucleus of the hypothalamus governs cyclical pineal
behavior.

Correct Answer. a

(18). Respiratory minute volume of lung is?

a. 4700 ml

b. 300 ml

c. 1200 ml

d. 6000 ml

Solution. Ans-18: (d) 6000 ml Ref.: Read the text below Sol :
The amount of air inspired per minute (pulmonary ventilation, respiratory minute volume) is normally about 6 L (500 ml/breath x 12
breaths/min).
The maximal voluntary ventilation (MVV) is the largest volume of gas that can be moved into and out of the lungs in 1 min by voluntary
effort. The normal MVV is 125 to 170 L/min.

Correct Answer. d

(19). Left shift in Arneth index indicates?

a. Anemia

b. Neutrophilia

c. Spleenomegaly

d. Hyperactive bone marrow

Solution. Ans-19: (b) Neutrophilia Ref.: Read the text below Sol :
The cook Arneth count or Arneth index describes the nucleus of a type of white blood cell called a neutrophil in an attempt to detect
disease.
Neutrophils typically have two or three lobes. In general, older neutrophils have more lobes than younger neutrophils.
The Arneth count determines the percentage of neutrophils with one(N1), two(N2), three (N3), four (N4), and five(N5) or more lobes.
The three lobed cell is mature and most efficient.
Individuals who have a large percentage of younger neutrophils with fewer lobes have a left shift (N1 + N2 + N3 > 80%) which can be
indicative of disease processes such as infection, malignant tumors, hemolytic crises, myocardial infarction, acidosis or hyperactive bone
marrow or leukamoid reaction.
Individuals with a larger percentage of older neutrophils with more lobes have a right shift (N4 + N5 > 20%) and most commonly
indicates hypo active bone marrow and have diseases such as vitamin B12 or folate deficiency, chronic uremia, liver disease, etc.

Correct Answer. b

Copyright 2014 Delhi Academy of Medical Sciences, All Rights Reserved. 8/76
(20). Diuretics, such as acetazolamide, which produce their effect by inhibiting carbonic anhydrase, inhibit the reabsorption of sodium in

a. The proximal tubule

b. The thick ascending limb of Henles loop

c. The distal convoluted tubule

d. The cortical collecting duct

Solution. Ans-20: (a) The proximal tubule Ref: Read the text below Sol :
Carbonic anhydrase is the enzyme that catalyzes the formation of CO2 and H2O from HCO3 and H+.
In the proximal tubule, the efficient reabsorption of bicarbonate requires the presence of carbonic anhydrase.
Carbonic anhydrase inhibitors prevent the formation of CO2 and therefore block the reabsorption of bicarbonate (and Na+), resulting in
a diuresis.
Since almost all of the filtered bicarbonate is reabsorbed in the proximal tubule, inhibiting carbonic anhydrase has little effect on
bicarbonate reabsorption from other segments of the nephron.

Correct Answer. a

(21). Which of the following is not considered a vascular source of nausea and vomiting ?

a. Headaches

b. Hypertension

c. Mesenteric angina

d. Congestive heart failure

Solution. Ans-21: (b) Hypertension Ref: Read the text below Sol :
Acute myocardial infarction classically causes nausea. Hypotension or congestive heart failure can cause a low-grade but persistent
nausea.
Mesenteric angina and the other choices cause nausea by diversion or diminishment of blood supply to the GI tract.
Headaches are also classic for nausea production due to the vascular involvement in the midbrain area.

Correct Answer. b

(22). In animals with chronic exposure to cold what is true

a. Sympathetice tone to heart is not much affected

b. Free insulin is increased

c. Vagal inhibition of heart is reduced

d. Perfusion of adipose tissue is decreased

Solution. Ans-22: (c) Vagal inhibition of heart is reduced Ref: Read the text below Sol : Endocrine glands in thermoregulation-
Increased catecholamine secretion is an important endocrine response to cold. Mice unable to make norepinephrine and epinephrine
because their hydroxylase gene is knocked out do not tolerate cold; they have deficient vasoconstriction and are unable to increase
thermogenesis in brown adipose tissue through UCP 1.
Thyroid and adrenal glands have significant roles in thermoregulation. Rats exposed to low temperatures (7-12 degrees Celsius) show
hyperplasia of the thyroid with metabolic rates increasing by as much as 16%. Thyroidectomized rats show little rise in metabolic rate
with similar cold exposure. Hypothalamic increase in TRH in response to cold with a resultant increase in TSH from the anterior pituitary
is responsible for thyroid responses to hypothermia.
Heat gain mechanisms activated on exposure to cold are:- Cutaneous vasoconstriction (the first response to occur) Horripilation
Non- shivering thermogenesis Refers to increased heat production due to increased cellular metabolism Includes Increased
sympathetic discharge increased secretion of catecholamines Uncoupling of oxidative phosphorylation which occurs in brown fat (in
adults this mechanism produces only 10-15% of the total heat) Increased secretion of thyroid hormones (doubtful role in humans)
Shivering Hunger Curling up Reduced air movement- staying indoors Increased voluntary activity Brown fat More abundant in
infants but present in adults as well Located in Between the scapulae Nape of neck Along the great vessels in the thorax and
abdomen, etc

Correct Answer. c

Copyright 2014 Delhi Academy of Medical Sciences, All Rights Reserved. 9/76
(23). CSF pressure is mainly regulated by

a. Rate of CSF formation

b. Rate of CSF absorption

c. Cerebral blood flow

d. Venous pressure

Solution. Ans-23: (b) Rate of CSF absorption Ref: Read the text below Sol :
Lumbar CSF pressure is 70 to 110 mms of water.
The rate of CSF formation is independent of intraventricular pressure. However, the rate of CSF absorption is proportional to the
pressure.
At a pressure of 112 mms of water, which is the average normal CSF pressure, filtration and absorption are equal. Below a pressure of
approximately 68 mms of water, absorption stops.
Large amounts of fluid accumulate and the CSF pressure increases when the
capacity for CSF absorption is decreased (external hydrocephalus, communicating hydrocephalus)

Correct Answer. b

(24). Compared with the resting state, during prolonged exercise, the caloric needs of skeletal muscle are met by

a. Release of free fatty acids from adipose tissue

b. An increase in hepatic glycogenolysis

c. An increase in gluconeogenesis in muscle

d. Increased intestinal uptake of glucose and amino acids

Solution. Ans-24: (b) An increase in hepatic glycogenolysis Ref: Read the text below Sol :
During prolonged exercise, skeletal muscle becomes relatively anaerobic when compared with its resting state and relies heavily upon
glycolysis for ATP production. The increase in glucose required is met primarily by glycogenolysis and gluconeogenesis in the liver.
Gluconeogenesis does not occur in skeletal muscle.
Thus, long-distance runners carbohydrate load before a race to increase hepatic glycogen stores. Intestinal uptake of glucose is
influenced primarily by dietary intake, which would not ordinarily increase during exercise.
At rest, the caloric needs of skeletal muscle are met primarily by mitochondrial oxidation of free fatty acids derived from adipose tissue.
Glucose is used primarily by the brain and erythrocytes.

Correct Answer. b

(25). Functions of the Sertoli cells in the seminiferous tubules include

a. Secretion of FSH into the tubular lumen

b. Secretion of testosterone into the tubular lumen

c. Maintenance of the blood-testis barrier

d. Synthesis of estrogen after puberty

Solution. Ans-25: (c) Maintenance of the blood-testis barrier Ref: Read the text below Sol :
The Sertoli cells rest on a basal lamina and form a layer around the periphery of the seminiferous tubules.
They are attached to each other by specialized junctional complexes that limit the movement of fluid and solute molecules from the
interstitial space and blood to the tubular lumen, and thus form a blood- testis barrier that provides an immunologically privileged
environment for sperm maturation.
Sertoli cells are intimately associated with developing spermatozoa and play a major role in germ-cell maturation.
They secrete a variety of serum proteins and an androgen-binding protein into the tubular fluid in response to FSH and testosterone
stimulation. Testosterone is synthesized and secreted by the interstitial Leydig cells.
Estrogen is produced in small amounts by the Sertoli cells before puberty.

Correct Answer. c

Copyright 2014 Delhi Academy of Medical Sciences, All Rights Reserved. 10/76
(26). Androgen binding protein is secreted by?

a. Pituitary

b. Liver

c. Sertoli cells

d. Leydig cells

Solution. Ans-26: (c) Sertoli cells Ref.: Read the text below Sol :
Androgen-binding protein (ABP) is a glycoprotein (beta-globulin) produced by the Sertoli cells in the seminiferous tubules of the testis
that binds specifically to testosterone (T), dihydrotestosterone (DHT), and 17-beta-estradiol.
Because ABP binds to T and DHT, these hormones are made less lipophilic and become concentrated within the luminal fluid of the
seminiferous tubules. The higher levels of these hormones enable spermatogenesis in the seminiferous tubules and sperm maturation in
the epididymis.

Correct Answer. c

(27). What will happen if one side of auditory cortex is removed?

a. Total hearing loss

b. Mild decrease in hearing

c. Decreased sound localization

d. Decreased sound interpretation

Solution. Ans-27: (c) Decreased sound localization Ref.: Read the text below Sol :
Destruction of both primary auditory cortices in the human being greatly reduces ones sensitivity for hearing. Destruction of one side
only slightly reduces hearing in the opposite ear; it does not cause deafness in the ear because of many crossover connections from side
to side in the auditory neural pathway.
However, it does affect ones ability to localize the source of a sound because comparative signals in both cortices are required for the
localization function.

Correct Answer. c

(28). Normal portal venous pressure is?

a. 5-10 mm Hg

b. 10-15 mm Hg

c. 15-20 mm Hg

d. 20-35 mm Hg

Solution. Ans-28: (a) 5-10 mm Hg Ref.: Read the text below Sol :
Normal portal venous pressure is 5-10 mm Hg.
A portal venous pressure minus IVC or Hepatic vein pressure difference determined by measuring hepatic vein pressure gradient
(HVPG)
Portal hypertension is defined as the elevation of the hepatic venous pressure gradient (HVPG) to > 5 mmHg.
Portal hypertension is characterized by increased resistance to flow in the portal venous system and sustained portal vein pressure
above 12 mm Hg.

Correct Answer. a

Copyright 2014 Delhi Academy of Medical Sciences, All Rights Reserved. 11/76
(29). Which of the following is not seen after removal of testis in an adult?

a. Loss of libido

b. Impotence

c. Muscle weakness

d. Decrease FSH

Solution. Ans-29: (d) Decrease FSH Ref.: Read the text below Sol :
Side effects of bilateral orchidectomy are secondary to androgen withdrawal (loss of libido, impotence, hot flushes, osteoporosis,
gynecomastria, muscle mass loss, anemia).
Testosterone and oestradiol normally feedbacks and inhibits GnRH, LH & FSH secretions.
Primary testicular (mumps, orchitis, orchidectomy) or primary ovarian failure (Turners syndrome, premature menopause) is associated
with very high LH and FSH levels.

Correct Answer. d

(30). Which does not pass from the superior cerebellar peduncle?

a. Dentatorubral tract

b. Anterior spinocerebellar tract

c. Posterior spinocerebellar tract

d. Tectocerebellar tract

Solution. Ans-30: (c) Posterior spinocerebellar tract Ref.: Read the text below Sol :
The decussation of superior cerebellar peduncle is the portion of the superior cerebellar peduncle which crosses into the midbrain. This
decussation comprises the cerebellothalamic tract or dentatothalamic tract and the cerebellorubral tract which arises from the globose
and emboliform nuclei to the contralateral red nucleus to become the rubrospinal tract.
The ventral spinocerebellar tract enters the cerebellum through the superior cerebellar peduncles, which otherwise mostly contain
efferent fibres.

Correct Answer. c

(31). Joint position sense from upper limb is carried by?

a. Dorsal spino cerebellar tract

b. Ventral spino cerebellat tract

c. Fasciculus cuneatus

d. Fasciculus gracilis

Solution. Ans-31: (c) Fasciculus cuneatus Ref.: Read the text below Sol :
The fasciculus cuneatus (tract of Burdach, named for Karl Friedrich Burdach) is a tract of nerves in the spinal cord that primarily
transmits information from the arms. It is part of the posterior column-medial lemniscus pathway.
The fasciculus cuneatus transmits fine touch, fine pressure, vibration, and proprioception information from spinal nerves located in
dermatomes C1 through T6.

Correct Answer. c

Copyright 2014 Delhi Academy of Medical Sciences, All Rights Reserved. 12/76
(32). Plasma membrane of cell is bonded to cytoskeleton by?

a. Ankyrin

b. Spectrin

c. Tubulin

d. Laminin

Solution. Ans-32: (a) Ankyrin Ref.: Read the text below Sol :
Spectrin, consists of two polypeptide chains, and which form intertwined (helical) flexible heterodimers. The head regions of
spectrin dimmers self-associate to form tetramers, while the tails associate with actin oligomers.
Eachactin oligomer can bind multiple spectrin tetramers, thus creating a two dimensional spectrin-actin skeleton that is connected to
the cell membrane by two distinct interactions.
The first, involving the proteins ankyrin and band 4.2, binds spectrin to the transmembrane ion transporter, band 3.
The second, involving band 4.1 binds the tail of spectrin to another transmembrane protein, glycophorin A

Correct Answer. a

(33). Coronary blood flow is maximum during?

a. Isovolumetric contraction

b. Rapid ejection

c. Slow ejection

d. Isovolumetric relaxation

Solution. Ans-33: (d) Isovolumetric relaxation Ref.: Read the text below Sol : Coronary Blood flow :
Coronary blood flow, like blood flow in other vessels, is dependent on a pressure gradient, principally driven by mean arterial pressure.
However, because aortic pressure can vary widely and because the heart beats continuously, the pressure gradient and, hence coronary
blood flow fluctuate depending on the state of contraction. During systole, especially on the left side of the heart, the pressure within the
coronary artery secondary to extravascular compression from the squeezing effect of the contracting myocardium virtually eliminates
antegrade coronary blood flow.
The coronary blood flow goes to zero just prior to ventricular ejection, corresponding to isovolumic contraction and increased
extravascular compression.
Conversely, coronary blood flow through the left side is maximal during early diastole, corresponding to the period of isovolumic
relaxation and minimal extravascular compression.
Coronary blood flow through the right side, however, is maximal during peak systole, because developed pressure and consequently
extravascular compression within the RV are considerably less than in the LV, thus allowing for antegrade flow during both systole and
diastole.

Correct Answer. d

Copyright 2014 Delhi Academy of Medical Sciences, All Rights Reserved. 13/76
(34). True abut receptor potential is?

a. Due to difference in permability of receptors for different ions

b. Is propogated

c. Refractory period of 15-20 sec

d. Is a graded change

Solution. Ans-34: (d) Is a graded change Ref.: Read the text below Sol :
Difference between Receptor potential and action potential
Receptor potential Action potential

Receptor potential is a graded response i.e. amplitude Action potential obeys all or none law i.e. further

of receptor potential increases with increase velocity of increase in stimulus threshold do not bring any change

stimulus application and increase strength of stimulus in amplitude

Can be added together if second stimulus arrives before Cannot be added tougher

the first stimulus is over

Has no refractory period Has a refractory period of 1 ms

Mostly it is local and cannot be propagated It can be propagated without loss in the amplitude

along the nerve fibre

Duration is greater (approximately 5-10 ms) Duration is small (approximately 1-2 ms)

Correct Answer. d

(35). Endometrium regeneration starts on which day of menstrual cycle?

a. 5th day

b. 7th day

c. 14th days

d. 28th day

Solution. Ans-35: (a) 5th day Ref.: Read the text below Sol :
Even during menses, increasing estrogen levels produced by the ovary begin the healing process, and the endometrium begins to
regenerate and prepare for the next cycle
The endometrial surface is re epithelialised within 4 to 7 days after the beginning of menstruation.
Ans-36: (d) An increase in serum progesterone levels Ref: Read the text below Sol :
Progesterone production by the corpus luteum increases significantly at the time of ovulation. Both LH and FSH blood levels increase
during the follicular phase of the menstrual cycle and reach peak blood levels prior to ovulation.
Estrogen levels follow a similar pattern during the follicular phase. The physiologic signal for ovulation is a surge in LH blood levels.
Under the influence of LH, thecal and granulosa cells become the luteal cells of the corpus luteum. Estrogen levels also increase, but do
not reach the levels achieved during the follicular phase.
Progesterone affects the set point for thermoregulation and increases body temperature approximately 0.5F.

Correct Answer. a

Copyright 2014 Delhi Academy of Medical Sciences, All Rights Reserved. 14/76
(36). An indication that ovulation has taken place is

a. An increase in serum FSH levels

b. A drop in body temperature

c. An increase in serum LH levels

d. An increase in serum progesterone levels

Solution. Ans-36: (d) An increase in serum progesterone levels Ref: Read the text below Sol :
Progesterone production by the corpus luteum increases significantly at the time of ovulation. Both LH and FSH blood levels increase
during the follicular phase of the menstrual cycle and reach peak blood levels prior to ovulation.
Estrogen levels follow a similar pattern during the follicular phase. The physiologic signal for ovulation is a surge in LH blood levels.
Under the influence of LH, thecal and granulosa cells become the luteal cells of the corpus luteum. Estrogen levels also increase, but do
not reach the levels achieved during the follicular phase.
Progesterone affects the set point for thermoregulation and increases body temperature approximately 0.5F.

Correct Answer. d

(37). The extracellular potassium of a hyperkalemic patient can be decreased by administering

a. Atropine

b. Epinephrine

c. Glucagon

d. Lactic acid

Solution. Ans-37: (b) Epinephrine Ref: Read the text below Sol :
The movement of K+ into cells is facilitated by the presence of insulin and epinephrine.
During exercise, epinephrine hastens the movement of K+ into muscle cells, preventing the accumulation of K+ in the extracellular
space around active muscle cells.
In cases of life-threatening hyperkalemia, insulin is often injected (along with glucose) to reduce K+ concentration.

Correct Answer. b

(38). Insulin regulates glucose transport into muscle and fat cells via which glucose transporter?

a. GLUT-1

b. GLUT-2

c. GLUT-3

d. GLUT-4

Solution. Ans-38: (d) GLUT-4 Ref: Read the text below Sol :
Following insulin binding on muscle and adipocytes, the glucose transporter GLUT-4 is translocated to the cell membrane where it
promotes glucose uptake by facilitated diffusion.
GLUT-1 transporters are found virtually everywhere, as are GLUT-3 transporters.
GLUT-2 transporters are found in liver, kidney, and intestinal cells. GLUT-5 transporters promote glucose
absorption from the jejunum.

Correct Answer. d

Copyright 2014 Delhi Academy of Medical Sciences, All Rights Reserved. 15/76
(39). A patient with hypercalcemia would be expected to experience

a. Diarrhea

b. Polyuria

c. Metabolic alkalosis

d. Euphoria

Solution. Ans-39: (b) Polyuria


Ref: Read the text below
Sol :
Hypercalcemia impairs renal function and leads to polyuria and polydipsia.
Hypercalcemia results from excessive bone resorption or increased calcium absorption from the small
intestine and is accompanied by inadequate renal excretion of calcium.
The disease also is associated with metabolic acidosis, constipation, lethargy, and anorexia

Correct Answer. b

(40). Citrate is a useful anticoagulant because of its ability to

a. Buffer basic groups of coagulation factors

b. Bind factor XII

c. Bind vitamin K

d. Chelate calcium

Solution. Ans-40: (d) Chelate calcium


Ref: Read the text below
Sol :
The citrate ion has three anionic carboxylate groups that avidly chelate calcium and reduce the concentration
of free calcium in blood.
Because free calcium (Ca2+) is required for multiple steps in both coagulation pathways, citrate is a useful
anticoagulant in vitro.
The citrate ion is rapidly metabolized; thus, blood anticoagulated with citrate can be infused into the body
without untoward effects. Oxalate, another calcium-chelating anticoagulant, is toxic to cells.

Correct Answer. d

Copyright 2014 Delhi Academy of Medical Sciences, All Rights Reserved. 16/76
(41). Turbulence is more likely to occur in a blood vessel if

a. The velocity of blood within the vessel increases

b. The viscosity of blood within the vessel increases

c. The diameter of the vessel decreases

d. The density of the blood decreases

Solution. Ans-41: (a) The velocity of blood within the vessel increases
Ref: Read the text below
Sol :
Osborne Reynolds described the critical factors affecting the flow of incompressible fluids in pipes late in the
nineteenth century. He discovered that the point at which flow changes from laminar (smooth) to turbulent is a
function of fluid density, viscosity, and velocity and of the diameter of the vessel expressed in the relationship
that became known as the Reynolds number (Re):

This relationship is equally valid for blood moving in the vessels of living organisms and for water moving in
pipes.
Increasing the length of the vessel may indirectly decrease the likelihood of turbulence by increasing vascular
resistance and thus decreasing blood velocity.

Correct Answer. a

(42). Angiotensin II causes all of the following except

a. Stimulation of thirst

b. Increased ADH secretion

c. Vasodilatation

d. Aldosterone secretion

Solution. Ans-42: (c) Vasodilatation


Ref: Read the text below
Sol :
Angiotensin II has the following actions
Stimulates thirst by a direct action on the sub fornicial organ (SFO)
Increases secretion of ADH and ACTH
Potent vasoconstrictor- produces arteriolar constriction and a rise in systolic and diastolic blood pressure
Increases aldosterone secretion by the adrenal cortex
Increases sodium reabsorption by a direct action on the thick ascending limb of loop of henle in the kidney
Causes contraction of mesangial cells which in turn causes 'kinking' of glomerular capillaries with a resultant
decrease in GFR
Causes both afferent and efferent arteriolar constriction, thereby decreasing GFR
Facilitates the release of nor epinephrine by a direct action on the post ganglionic sympathetic neurons
Decreases the sensitivity of the baroreceptors- this potentiates the pressor effect of angiotensin
Acts on area postrema which potentiates the pressor effects of angiotensin

Correct Answer. c

Copyright 2014 Delhi Academy of Medical Sciences, All Rights Reserved. 17/76
(43). Which of the following statements regarding cutaneous shunt vessels is true

a. Perform nutritive function

b. Have a role in thermoregulation

c. Not under the control of autonomic nervous sytem

d. These vessels are evenly distributed throughout the skin

Solution. Ans-43: (b) Have a role in thermoregulation Ref: Read the text below Sol :
Arteriovenous (A-V) anastomoses or shunt vessels are short channels connecting arterioles to venules bypassing the capillaries.
They have thick, muscular walls and are abundantly innervated by vasoconstrictor nerves.
They are located in the fingers, palms and ear lobes. These vessels have a role to play in thermoregulation

Correct Answer. b

(44). Which of the following statements about the third heart sound (S3) is correct?

a. It is usually diminished in congestive heart failure

b. It is produced by turbulence during rapid ventricular filling in early diastole

c. It is produced by turbulence following atrial contraction

d. It is often associated with the floppy mitral valve syndrome

Solution. Ans-44: (b) It is produced by turbulence during rapid ventricular filling in early diastole Ref: Read the text below Sol :
The third heart sound (S3) is commonly heard in children and young adults. It is usually a pathological sign when heard in adults over
40.
It is produced by the sudden deceleration of blood at the end of the rapid ventricular filling phase of the cardiac cycle.
It is most often caused by conditions that decrease ventricular compliance, such as increased end-systolic volumes, ischemic heart
disease, or cardiac tamponade.
However, it can also occur when rapid filling of the ventricle is abnormally high, for example, in anemia or mitral regurgitation

Correct Answer. b

(45). In a resting, healthy man, the ejection fraction is approximately?

a. 0.1

b. 0.2

c. 0.3

d. 0.6

Solution. Ans-45: (d) 0.6 Ref: Read the text below Sol :
The ejection fraction is the fraction of blood ejected from the ventricle during systole, which is typically between 0.6 and 0.7.
Ejection fraction can be increased by an increase in contractility or a decrease in afterload.
Heart failure causes a decrease in ejection fraction.

Correct Answer. d

Copyright 2014 Delhi Academy of Medical Sciences, All Rights Reserved. 18/76
(46). Sustained elevation of cardiac output will occur with which of the following conditions?

a. Hypertension

b. Aortic regurgitation

c. Anemia

d. Third-degree heart block

Solution. Ans-46: (c) Anemia Ref: Read the text below Sol :
The magnitude of the cardiac output is regulated to maintain an adequate blood pressure and to deliver an adequate supply of oxygen to
the tissues.
In anemia, a greater cardiac output is required to supply oxygen to the tissues because the oxygencarrying capacity of the blood is
reduced. In aortic regurgitation, the stroke volume will be increased.
However, a portion of the blood ejected by the heart will return to the heart during diastole.
Thus, the output delivered to the tissues does not increase despite the fact that the blood ejected by the heart has increased.
In hypertension, third-degree heart block, and cardiac tamponade (decreased filling of the heart due to accumulation of fluid within the
pericardium), cardiac output will be normal, or, if compensation is not possible, cardiac output will be reduced.

Correct Answer. c

(47). Low-fluid-volume states

a. Inhibit ADH

b. Inhibit the rennin-angiotensin system

c. Produce bradycardia

d. Stimulate sodium retention

Solution. Ans-47: (d) Stimulate sodium retention Ref: Read the text below Sol :
The kidneys respond to low-volume states by releasing ADH, which increases water reabsorption, and by activating the rennin-
angiotensin system to release aldosterone, which stimulates sodium retention in the tubules.
Low volume states are associated with tachycardia and a reduced effective circulatory volume.

Correct Answer. d

(48). Which of the following is not seen in hyponatremia ?

a. Confusion

b. Lethargy

c. Muscle twitching

d. Paralysis

Solution. Ans-48: (d) Paralysis Ref: Read the text below Sol :
Hyponatremia is associated with increased muscle irritability, confusion, and possible seizure activity.
Lethargy is common, especially in light of dehydration.
Paralysis is more commonly seen in hypernatremic states.

Correct Answer. d

Copyright 2014 Delhi Academy of Medical Sciences, All Rights Reserved. 19/76
(49). Which of the following would cause an immediate reduction in the amount of potassium leaking out of a cell?

a. Increasing the permeability of the membrane to potassium

b. Increasing the intracellular potassium concentration

c. Increasing (hyperpolarizing) the membrane potential

d. Reducing the activity of the sodium-potassium pump

Solution. Ans-49: (c) Increasing (hyperpolarizing) the membrane potential Ref: Read the text below Sol :
The amount of potassium leaking out of the cell depends on its driving force and its membrane conductance.
The driving force is the difference between the membrane potential and the equilibrium potential for potassium.
Since the membrane potential is more positive than the equilibrium potential for potassium, hyperpolarizing the membrane (that is,
making it more negative) would reduce the driving force.
Increasing the intracellular potassium concentration increases the equilibrium potential for potassium, that is, makes it more negative
and thus increases the driving force. Reducing the sodiumpotassium pump activity would cause the potassium concentration gradient to
fall, which, in turn, would cause a decrease in the amount of potassium leaking out of the cell.
However, this would not occur immediately. In fact, because the pump is electrogenic, a small depolarization would initially follow a
cessation of pump activity and this would cause an increase in the driving force for potassium.

Correct Answer. c

(50). In 17-hydroxylase deficiency, which hormone is not affected?

a. Testosterone

b. Estrogen

c. Corticosterone

d. Cortisol

Solution. Ans-50: (c) Corticosterone Ref.: Read the text below Sol :
In 17 - hydroxylase deficiency, due to mutated gene for CYP17, no sex hormones are produced, so female external genitalia are present
in males.
However, the pathway leading to corticosterone and aldosterone is intact the elevated levels of which produces hypertension and
hypokalemia.
Cortisol is deficient, but this is partially compensated by glucocorticoid activity of corticosterone.

Correct Answer. c

(51). All of the following are false about narcolepsy except:

a. REM sleep latency is increased

b. Defective NREM sleep regulation

c. Catalepsy is often associated with narcolepsy

d. Associated with HLA-DR2

Solution. Ans-51: (d) Associated with HLA-DR2 Ref.: Read the text below Sol : Narcolepsy
Narcolepsy is a disorder of defective REM sleep regulation, with decreased REM sleep latency
The classic Narcolepsy tetrad consist of :
Excessive day time somnolence
Sudden weakness or loss of muscle tone without loss of consciousness, often elicited by emotion (cataplexy)
Hallucinations at sleep onset (hypnagogic hallucinations) or upon awakening (hypnapompic hallucinations)
Muscle paralysis upon awakening (sleep paratysis)
Associated with HLA-DR2 or HLA-DQB1
Narcolepsy is characterized by a sudden onset of REM sleep.
Hypocretin (orexin) producing neurons in the brain is reduced.

Correct Answer. d

Copyright 2014 Delhi Academy of Medical Sciences, All Rights Reserved. 20/76
(52). A ventrolateral cordotomy relieving pain in right leg is effective because it interrupts the:

a. Left dorsal column

b. Left ventral spinothalamic tract

c. Left lateral spinothalamic tract

d. Right lateral spinothalamic tract

Solution. Ans-52: (c) Left lateral spinothalamic tract Ref.: Read the text below Sol :
To produce relief of pain in leg the contrlateral lateral spinothalamic tract should be cut or interrupted.
Anterolateral cordotomy is effective in relieving pain because it cuts/interrupts lateral (not anterior) spinothalamic tract carrying pain
(and temperature) from the opposite side of body.
This is the reason why, to produce relief of pain in right leg the CL (i.e. left) lateral spinothalamic tract is cut or interrupted.

Correct Answer. c

(53). Decrease extracellular calcium leads to?

a. Decrease excitability

b. Decrease membrane stability

c. Increases amount of depolarization needed

d. All of the above

Solution. Ans-53: (b) Decrease membrane stability Ref.: Read the text below Sol :
A decrease in extracellular Ca2+ concentration increases the excitability of nerve and muscle cells by decreasing the amount of
depolarization necessary to initiate the changes in the Na+ and K+ conductance that produce the action potential.
Conversely, an increase in extracellular Ca2+ concentration stabilizes the memberane by decreasing excitability.

Correct Answer. b

(54). Regarding caissons disease which statement among the following is correct?

a. Lung damage is caused by air embolism

b. Pain in the joints is due to nitrogen bubbles

c. Tremors are seen due to nitrogen narcosis

d. High pressure Nervous syndrome can be prevented by using mixtures of Oxygen & Helium

Solution. Ans-54: (b) Pain in the joints is due to nitrogen bubbles Ref.: Read the text below Sol : Decompression sickness :
As a diver breathing 80% N2 ascends from a dive, the elevated alveolar PN2 falls. N2 diffuses from the tissues into the lungs along the
partial pressure gradient. If the return to atmospheric pressure (decompression) is gradual, no harmful effects are observed; but if the
ascent is rapid, N2 escapes from solution.
Bubbles form in the tissues and blood, causing the symptoms of decompression sickness (the bends, caisson disease). Bubbles in the
tissues cause severe pains, particularly around joints, and neurologic symptoms that include paresthesias and itching.

Correct Answer. b

Copyright 2014 Delhi Academy of Medical Sciences, All Rights Reserved. 21/76
(55). Which among the following is a features of enervation of smooth muscle?

a. Atrophy of the muscle.

b. Decrease in no of receptors for neurotransmitters

c. Increased sensitivity to chemical mediators

d. Decreased neurotransmitter release at NMJ

Solution. Ans-55: (c) Increased sensitivity to chemical mediators Ref.: Read the text below Sol :
When the motor nerve supply to a smooth muscle is cut, the muscle does not atrophy (this is in contrast to skeletal muscle which
atrophies on denervation, it becomes hyper responsive to the chemical mediators that normally activate it.

Correct Answer. c

(56). Normal function of peritoneum is all except :

a. Fibrinolytic

b. Peritoneal fluid provides nourishment of Gut

c. Free movement of viscera

d. Removes excess fluid and particulates

Solution. Ans-56: (b) Peritoneal fluid provides nourishment of Gut Ref.: Read the text below Sol : Functions of peritoneum are :
Both secretion and absorption of peritoneal fluid (fluid exchange with the extracellular fluid space at rates of 500 ml. or more per hour)
The fluid lubricates the visceral peritoneum and allows the mobile viscera to glide freely on the abdominal wall and each other within
the limits dictated by their attachments
Peritoneal fluid contains water, proteins, electrolytes and solutes derived from interstitial fluid in the adjacent tissues and from the
plasma in the local blood vessles. It normally contains a flew cells, including desquamated mesothelium, peritoneal macrophages, mast
cells, fibroblasts, lymphocytes and other leukocytes.
Lymphocytes provide both cellular and humoral immunological defense mechanisms within the peritoneal cavity.
Complement mediated antibacterial activity
Particulate matter, including bacteria up to 20 mm in size is cleared via stomas in the diaphragm.
Normally there is less than 50 ml of free peritoneal fluid, a transudate with the following characteristics; specific gravity below 1.016,
protein concentration less than 3 g/dL, white blood cell count less than 3000/L, complement mediated antibacterial activity, and lack of
fibrinogen related clot formation.

Correct Answer. b

(57). Sperms are released during ejaculation from :

a. Seminal vesicles

b. Epididymis

c. Testes

d. Prostate

Solution. Ans-57: (b) Epididymis Ref.: Read the text below Sol :
Sperms are formed in seminiferous tubules and stored in epididymis till ejaculation.
The sequence of sperm movement from within the testis to urethra is as follows.
Seminiferous tubules straight tubule Rete testis Efferent tubule Epididymis Vas deferens Ejaculatory ducts Urethra

Correct Answer. b

Copyright 2014 Delhi Academy of Medical Sciences, All Rights Reserved. 22/76
(58). Slowest conduction velocity is seen in which nerve fiber?

a. A-alpha

b. A-beta

c. A-delta

d. C fiber

Solution. Ans-58: (d) C fiber Ref.: Read the text below Sol :
Relative susceptibility of Mammalian A, B, and C nerve Fibres to conduction block Produced by Various Agents.
Susceptibility to: Most susceptible Intermediate Least Susceptible

Hypoxia B A C

Pressure A B C

Local anaesthetics C B A

Correct Answer. d

(59). All of the following undergo cell division except :

a. Pericyte

b. Cardiac muscle cell

c. Smooth muscle cell

d. Satellite cell of skeletal muscle

Solution. Ans-59: (b) Cardiac muscle cell Ref.: Read the text below Sol :
It is generally accepted that binucleated cardiomyocytes cannot divide, but can readily enlarge
Whereas the majority of cardiomyocytes are terminally differentiated and possess no regenerative capacity, a small number of precursor
cells do exist and these retain the ability to divide and from mature cardiomyocytes
Until recently, it was commonly believed that cardiac muscle cells could not be regenerated.

Correct Answer. b

(60). Wolf-Chaikoff effect is seen due to?

a. Inhibition of cAMP response of TSH

b. Inhibition of proteolysis of Thyroglobulin

c. Inhibition of organic binding of iodide

d. Inhibition of iodide trapping mechanism.

Solution. -NA-

Correct Answer. c

Copyright 2014 Delhi Academy of Medical Sciences, All Rights Reserved. 23/76
(61). Which gas is used to measure diffusion capacity

a. CO

b. Nitrogen

c. CO2

d. None

Solution. Ans-61: (a) CO Ref: Ganong 23rd edi pg 601-602; Guyton11th edition pg 499 Sol:
o The diffusing capacity of the lungs is directly proportional to the surface area available for diffusion and the thickness of the
respiratory membrane.
o The diffusing capacity for CO (DLCO) is measured as an index of diffusing capacity of the lungs.
o DLCO is proportionate to the amount of CO entering the lungs divided by the partial pressure of CO in the alveolus minus the partial
pressure of CO in the blood in the pulmonary capillaries. Since the latter term is almost zero it can be ignored and the equation now
becomes:
o DLCO = VCO>/ PACO
o The normal value of DLCO at rest is about 25 mL/ min/ mm of Hg (Guyton- 17 mL/ min/ mm of Hg). It increases up to threefold during
exercise because of capillary dilatation and an increase in the number of active capillaries.
o Oxygen is directly not used for measuring the diffusing capacity because, as is obvious from the above description of measurement of
DLCO a sample of blood from the pulmonary capillaries will be required to measure
the partial pressure of oxygen in the pulmonary capillaries (this is being ignored while measuring DLCO since the partial pressure of CO
in the pulmonary capillaries is close to zero).
o To convert DLCO to DLO2 the value of DLCO is multiplied by a factor of 1.23 because the diffusion coefficient for oxygen is 1.23 times
that for CO.

Correct Answer. a

(62). Destruction of the supraoptic nuclei of the brain will produce which of the following changes in urinary volume and concentration?
(Assume that fluid intake equals fluid loss.)

a. An increased urinary volume and a very dilute urine

b. An increased urinary volume and concentrated urine

c. A normal urinary volume and concentration

d. Decreased urinary volume and a very dilute urine

Solution. Ans-62: (a) An increased urinary volume and a very dilute urine Ref: Read the text below Sol:
Destruction of the supraoptic nuclei eliminates antidiuretic hormone (ADH) production.
In the absence of ADH, the collecting ducts of the kidney are impermeable to water and therefore all of the fluid passing through the
distal convoluted tubule, about 10 to 20 L of hypotonic fluid per day, is excreted in the urine.

Correct Answer. a

(63). Which of the following substances will be more concentrated at the end of the proximal tubule than at the beginning of the proximal
tubule?

a. Glucose

b. Creatinine

c. Sodium

d. Bicarbonate

Solution. Ans-63: (b) Creatinine Ref: Read the text below Sol:
Sodium is isosmotically reabsorbed from the proximal tubule; that is, when sodium is reabsorbed,water flows out of the proximal tubule
to maintain a constant osmolarity.
Thus, the concentration of sodium does not normally change as the filtrate flows through the proximal tubule.
Since creatinine cannot be reabsorbed from the tubule, its concentration rises as water is reabsorbed.
The concentrations of glucose, bicarbonate, and phosphate are all less at the end of the proximal tubule than at the beginning.

Correct Answer. b

Copyright 2014 Delhi Academy of Medical Sciences, All Rights Reserved. 24/76
(64). What is the clearance of a substance, if its concentration in plasma in 10 mg%, concentration in urine is 100 mg% and urine flow is 2
ml/min :

a. 0.2 ml/min

b. 0.2 ml/min

c. 2 ml/min

d. 20 ml/min

Solution. Ans-64: (d) 20 ml/min Ref.: Read the text below Sol :
Clearance of a substance, if its concentration in plasma in 10 mg% concentration in urine is 100 mg% and urine flow is 2 ml/min is 20
ml/min.
Clearance
UV 100 2

Clearance = = . = 20 ml/min
P 10

Correct Answer. d

(65). The refractory power of eye is normally 59 diopters. Two thirds of this is contributed by :

a. Anterior surface of the lens

b. Posterior surface of the lens

c. Anterior surface of the cornea

d. Aqueous humor and vitreous humor combined

Solution. Ans-65: (c) Anterior surface of the cornea Ref.: Read the text below Sol :
The refractory power of eye is normally 59 diopters.
Two thirds (not exactly, it is 3/4th) of this is contributed by Anterior surface of the cornea.

Correct Answer. c

(66). The positive chronotropic action of epinephrine is due to all of the following effects on cardiac action potential, except :

a. Increase in the rate of phase zero

b. Increase in the rate of phase one

c. Shortening of phase two

d. Increase in the rate of repolarization of phase three

Solution. Ans-66: (c) Shortening of phase two Ref.: Read the text below Sol : The positive chronotropic action of epinephrine is due to all
of the following effects on cardiac action potential, except shortening of phase two.
Nor-epinephrine secreted by sympathetic nerve endings and epinephrine exert their chronotropic action by acting on SA node.
The action potential at the SA node is known as the pacemaker potential.
The pacemaker potential has only tree phases phase 0, phase 1 and phase 4.

Correct Answer. c

Copyright 2014 Delhi Academy of Medical Sciences, All Rights Reserved. 25/76
(67). The graph in Fig shows the jugular vein pressure curve of this patient. If the first heart sound occurs between letters C and D, which part
of the curve represents the atrial contraction?

a. Peak A

b. Peak c

c. Peak d

d. Trough B

Solution. Ans 67: (b) Peak C Reference: Read the text below Sol :
Fig shows the jugular vein pressure curve. Pressure in the jugular vein reflects atrial pressure and is highest during the atrial
contraction (named the a wave).
The v wave represents the rise in atrial pressure before the tricuspid valve opens during diastole.
The y notch is due to the fall in atrial pressure during the ventricular filling phase.
The c wave is due to the rise in atrial pressure produced by the bulging of the tricuspid valve during isovolumetric contraction of the
right ventricle.
The x notch coincides with the ventricular ejection phase

Correct Answer. b

(68). A patient with untreated diabetes mellitus is admitted to the hospital for treatment. Laboratory findings include a blood pH below 7.2.
Which of the following is likely to directly result from this level of acidemia?

a. Arteriolar constriction

b. Decreased catabolism

c. Hyperkalemia

d. Hypoventilation

Solution. Ans 68: (c) Hyperkalemia Reference: Read the text below Sol :
Acidemia is the presence of excess H ions in blood, which tends to create a rise in the extracellular potassium concentration.
The rule of thumb for an inorganic acid that causes the acidity is that a decrease in pH of 0.1 unit leads to a rise in plasma potassium of
0.6 milliequivalents per liter.
The effect of an organic acid is a lot less. The mechanism responsible for the relationship between pH and extracellular potassium
concentration is not completely understood.
One effect is that the intracellular acidosis inhibits K influx by reducing the activity of the Na-K-pump.
An exchange of extracellular H for intracellular K might also contribute.

Correct Answer. c

Copyright 2014 Delhi Academy of Medical Sciences, All Rights Reserved. 26/76
(69). Umblical vein carries?

a. Oxygenated blood towards placenta

b. Oxygenated blood away from placenta

c. Deoxygenated blood towards placenta

d. Deoxygenated blood away from placenta

Solution. Ans-69: (b) Oxygenated blood away from placenta Ref.: Read the text below Sol :
During prenatal development, the umbilical cord is physiologically and genetically part of the fetus and (in humans) normally contains
two arteries (the umbilical arteries) and one vein (the umbilical vein), buried within Whartons jelly.
The umbilical vein supplies the fetus with oxygenated, nutrientrich blood from the placenta. Conversely, the fetal heart pumps
deoxygenated, nutrient-depleted blood thorough the umbilical arteries back to the placenta.

Correct Answer. b

(70). Insulin increase glucose uptake in?

a. Small intestine

b. RBC

c. Skeletal muscle

d. Kidney

Solution. Ans-70: (c) Skeletal muscle Ref.: Read the text below Sol :
Insulin dependent glucose transportation in mediated by GLUT4 which is expressed in skeletal muscles, cardiac muscles and adipose
tissue
Transporter Function Site

GLUTI 1 Basal glucose uptake RBC, kidney, placenta, BBB

GLUTI 2 Beta cell glucose sensor, transporter out of renal Beta cell, small intestine, kidney

and intestinal epithelial cells

GLUTI 3 Basal glucose uptake Brain, placenta, kidney

GLUTI 4 Insulin stimulated glucose uptake Skeletal and cardiac muscle and adipose

tissue

GLUTI 5 Fructose transporter Jejunum, sperm

GLUTI 6 None Pseudogene

GLUTI 7 Glucose 6PO4 transporter in ER Liver

GLUTI 1 Active uptake of glucose against conc. Gradient Small I intestine, renal tubule

GLUTI 2 Renal tubule

Correct Answer. c

Copyright 2014 Delhi Academy of Medical Sciences, All Rights Reserved. 27/76
(71). Predominant site of erythropoiesis during 6th month of gestation is?

a. Yolk sac

b. Liver

c. Bone marrow

d. Thymus

Solution. Ans-71: (b) Liver Ref.: Read the text below Sol : Yolk sac hemopoiesis
During prenatal development, the sites of hematopoiesis change several times.
In humans and other vertebrates, the first hematopoietic cells arise during late gastrulation in the extraembryonic yolk sac in structures
known as blood islands.
This initial hematopoiesis is termed primitive hematopoiesis.
Hematopoiesis is first established soon after implantation of the blastocyst, with the appearance of primitive erythroid cells in blood
islands of the yolk sac beginning at day 18 of gestation. (Williams)
Primitive hematopoiesis is transient, occurring on embryonic day 19 through week 8 in humans.
After 7 weeks gestation, hematopoietic progenitors are no longer detected in the yolk sac.

Correct Answer. b

(72). Gammaglobulins are formed by?

a. B cells

b. T cells

c. Plasma cells

d. Liver

Solution. Ans-72: (c) Plasma cells Ref.: Read the text below Sol : On the basis of electrophoresis Globulins are classified as :
Alpha 1 and 2 globulins : e.g. Glycoproteins and hormone binding globulins
Beta globulins :
Gamma globulins : immunoglobulins are a type of gamma globulins and they are produced by plasma cells.

Correct Answer. c

(73). What happens when carotid sinus is pressed?

a. Heart rate decreases, peripheral resistance increases

b. Heart rate and peripheral resistance decreases

c. Peripheral resistance and contractility decreases

d. Peripheral resistance and contractility increases

Solution. Ans-73: (b) Heart rate and peripheral resistance decreases Ref.: Read the text below Sol : Carotid Sinus Pressure :
The most commonly used and most effective vagal maneuver in terminating SVT is carotid sinus pressure.
It should be performed under cardiac monitoring in recumbent position.
With neck hyperextended the common carotids is identified by its pulsations and followed distally as close to the mandible as possible,
usually to the angle of mandible, where the common carotid bifurcates. It is the bifurcation where carotid sinus is located.
Carotid sinus pressure will produce increased baroreceptor discharge inhibits the tonic discharge of the vasoconstrictor nerves and
excites the vagal innervations of the heart, producing vasodilation, venodilation a drop in blood pressure, bradycardia, and a decrease in
cardiac output.
Carotid pressure is applied gently and constantly using both middle and index finger.
The maneuver canbe repeated several times until the response is elicited but should never exceeds more than 5 seconds
It is contraindicated in carotid stenosis.

Correct Answer. b

Copyright 2014 Delhi Academy of Medical Sciences, All Rights Reserved. 28/76
(74). Satiety center is located at?

a. Ventromedial nucleus

b. Lateral hypothalamus

c. Supra median nucleus

d. Preoptic nucleus

Solution. Ans-74: (a) Ventromedial nucleus Ref.: Read the text below Sol :
Hypothalamic regulation of the appetite for food depends primarily on the interaction of two areas: a lateral feeding center in the bed
nucleus of the medial forebrain bundle at its junction with the pallidohypothalamic fibers and medial satiety center in the ventromedial
nucleus.

Correct Answer. a

(75). Band not covered by actin filament is?

a. H band

b. I band

c. M band

d. Z band

Solution. Ans-75: (a) H band Ref.: Read the text below Sol :
The area between two adjacent Z lines is called a sarcomere
The thick filaments, which are about twice the diameter of the thin filaments, are made up of myosin; the thin filaments are made up of
actin, tropomyosin and troponin.
The thick filaments are lined up to form the A bands, whereas the array of thin filaments forms the less dense I bands.
The lighter H bands in the center of the A bands are the regions where, when the muscle is relaxed, the thin filaments do not overlap the
thick filaments.
The Z lines transect the fibrils and connect to the thin filaments. If a transverse section through the A band is examined under the
electron microscope, each thick filament is seen to be surrounded by six thin filaments in a regular hexagonal pattern.

Correct Answer. a

(76). Iodine uptake into thyroid gland is an example of?

a. Primary active transport

b. Secondary active transport

c. Facilitated diffusion

d. Endocytosis

Solution. Ans-76: (b) Secondary active transport Ref.: Read the text below Sol :
The thyroid cell membranes facing the capillaries contain symporter, or iodide pump, that transports Na+ and I- into the cells against
the electrochemical gradient for I-
This Na+/I- symporter (NIS) is capable of producing intracellular I- concentrations that are 20-40 times as great as the concentration in
plasma. The process involved is secondary active transport.

Correct Answer. b

Copyright 2014 Delhi Academy of Medical Sciences, All Rights Reserved. 29/76
(77). All are present in dense granules of platelets except:

a. ADP

b. vWF

c. Calcium

d. Serotonim

Solution. Ans-77: (b) vWF Ref.: Read the text below Sol : Platelet has
Dense granules : contain the non-protein substances that are secreted in response to platelet activation, including serotonin, ADP, and
other adenine nucleotides
Alpha granules : contain secreted proteins other than the hydrolases in lysosomes. These proteins include clotting factors and platelet-
derived growth factor (PDGF).
Platelet membranes contain receptors for collagen, ADP, vessel wall von willebrand factor, and fibrinogen

Correct Answer. b

(78). The change of which of the following substances is most commonly monitored as indicator for the efficiency of blood glucose control?

a. Hemoglobin A1c (HbA1c)

b. Lipoprotein

c. Modified albumin

d. Myoinositol

Solution. Ans 78: (a) Hemoglobin A1c (HbA1c) Reference: Read the text below Sol :
The hallmark of poorly controlled diabetes mellitus is elevated blood glucose, which causes unphysiological glycosylation of proteins.
An important long-term measure of blood glucose control in patients with diabetes mellitus is to monitor the modification of hemoglobin
A to form glycated hemoglobins. HbA1c is the major subfraction, and determination of HbA1c is usually achieved by ion-exchange or gel
electrophoresis.
The level of glycated hemoglobins in the blood is directly related to the average blood glucose levels over the life span of the hemoglobin
in the circulation. Since the half-life of red blood cells is about 120 days, a single determination of glycated hemoglobin reflects the
average blood glucose level during the preceding 812 weeks.
Lipoprotein (a) is a lipoprotein particle implicated in atherosclerosis and thrombosis. Albumin is the most abundant plasma protein, but
is not significantly affected by glycosylation.
Myoinositol is a signaling molecule the decrease in response of which to elevated sorbitol has been suggested as a complication of
diabetes.
Sorbitol is another sugar derivative, unrelated to HbA1c that is believed important in causing other diabetic complications such as
cataracts and peripheral neuropathy.

Correct Answer. a

Copyright 2014 Delhi Academy of Medical Sciences, All Rights Reserved. 30/76
(79) Which of the following proteins does not cause secretion of gastric acid
. ?

a. Acetylcholine

b. Somatostatin

c. Histamine

d. Gastrin

Solution. Ans-79: (b) Somatostatin Ref: Bailey and Loves - 1047

Stimuli increasing gastric acid secretion


1. Gastrin released by G cells in response to food in stomach.
2. Histamine (most important) produced by ECL (enterochromaffin-
like cell) in response to gastrin.
3. Acetylcholine Produced in response to distension of stomach.

Correct Answer. b

(80). Thrombomodulin-thrombin complex prevents clotting because :

a. Thrombomodulin inhibits prothrombin activator

b. Thrombomodulin-thrombin complex activities on antithrombin III

c. The binding of thrombomodulin with thrombin removes thrombin and also activities a plasma protein C which inactivates activated
factors V & VIII

d. Thrombomodulin-thrombin complex activates heparin.

Solution. Ans-80: (c) The binding of thrombomodulin with thrombin removes thrombin and also activities a plasma protein C which
inactivates activated factors V & VIII Ref.: Read the text below Sol :
Thrombomodulin-thrombin complex prevents clotting because binding of thrombomodulin with thrombin removes thrombin and also
activities a plasma protein C which inactivates activated factors V & VIII.

Correct Answer. c

Copyright 2014 Delhi Academy of Medical Sciences, All Rights Reserved. 31/76
(81). If the blood flow through a blood vessel is 2 ml/min, how much will it become if its radius is doubled :

a. Will remain the same

b. 4 ml/min

c. 8 ml/min

d. 16 ml/min

Solution. Ans-81: (d) 16 ml/min Ref.: Read the text below Sol :
If the blood flow through a blood vessel is 2 ml/min, it will become 16 ml/min if its radius is doubled.
8L
R = .
4
r
R = resistance, r = radius of vessel

So resistance decreases around 16 times, when radius is doubled, hence blood flow also increases 16 times.

Correct Answer. d

(82). Iron is absorbed actively in the:

a. Stomach

b. Duodenum

c. Terminal ileum

d. Proximal ileum

Solution. Ans:82: (b) Duodenum Ref: Ganong 22nd/477 Sol:


Iron Absorption
1. Almost all iron absorption occurs in duodenum and upper jejunum (proximal small intestine)
3+ 2+
3. Most of the iron is in ferric form (Fe ) but it is ferrous form (Fe ) that is absorbed

Correct Answer. b

(83). Major neurotransmitter released at end organ effector of the sympathetic division of the autonomic nervous system:

a. Adrenaline

b. Noradrenalin

c. Dopamine

d. Acetylcholine

Solution. Ans:83: (b) Noradrenalin Ref: Ganong 22nd/223-224 Sol:


The terminal nerve endings of the parasympathetic system all or virtually all secrete acetylcholine
Almost all of the sympathetic nerve endings secrete nor-epinephrine, but a few secrete acetylcholine.
These hormones in turn act on the different organs to cause respective parasympathetic or sympathetic effects.
Therefore, acetylcholine is called parasympathetic transmitter and nor-epinephrine is called sympathetic transmitter.

Correct Answer. b

Copyright 2014 Delhi Academy of Medical Sciences, All Rights Reserved. 32/76
(84). All of the following statements about action potentials are true except

a. The amptitude is large (70-110mv)

b. The duration is brief (1-10 msec)

c. It has a graded response

d. Its propagation is active (self-propagating)

Solution. Ans-84: (c) It has a graded response Ref:Read the text below Sol:
An action potential is a self-propagating spike response (all-or-none response) unique to excitable tissues.
Voltage-sensitive sodium-ion (Na+) and potassium-ion (K+ channels distinct from the K+ - Na+ leak channels underlie the molecular basis
of the action potential.
Compared to local potentials, action potentials have a large amplitude (70-110mv) and a brief duration (1-10 msec).

Correct Answer. c

(85). Glomerular filtration Rate is increased when

a. Plasma oncotic pressure is increased

b. Glomerular hydrostatic pressuire is decreased

c. Tubular hydrostatic pressure is decreased

d. Increased renal blood flow

Solution. Ans:85: (d) Increased renal blood flow Ref: Ganong 22nd/707, 708 Sol:
Glomerular filtration rate is increased in increased renal blood flow.

Correct Answer. d

(86). Which of the following product is primarily filtered with little secretion or re-absorption in renal tubules?

a. Sodium

b. Creatinine

c. Glucose

d. Amino acids

Solution. Ans:86: (b) Creatinine Ref: Ganong 22nd/706 Sol: GFR Measurement
GFR can be measured by measuring the excretion & plasma level of a substance that is freely filtered through glomeruli and neither
secreted nor absorbed by the tubules.Characteristics of substances used to measure GFR
Fitered freely
Neither reabsorbed, nor secreted by tubules
Non-toxic
Not metabolized by body
Substances used
Inulin (standard)
Creatinine

Correct Answer. b

Copyright 2014 Delhi Academy of Medical Sciences, All Rights Reserved. 33/76
(87). All of the following effects can occur if vagus is stimulated, except :

a. Reduction in blood pressure

b. Increase in secretions of the intestine

c. Reduction in heart rate

d. Bronchial musculature relaxation

Solution. Ans-87: (d) Bronchial musculature relaxation Ref: Ganongs Physiology, 23rd ed., p.-238 Sol :
Activation of the vagus nerve typically leads to a reduction in heart rate, blood pressure, or both.
This occurs commonly in the setting of gastrointestinal illness such as viral gastroenteritis or acute cholecystitis, or in response to other
stimuli, including carotid sinus massage, Valsalva maneuver, or pain from any cause, in particular, having blood drawn. When the
circulatory changes are great enough, vasovagal syncope results. Relative dehydration tends to amplify these responses.

Correct Answer. d

(88). True statement regarding alpha wave is :

a. They are often slightly higher in amplitude on the dominant side

b. Alpha activity appears normally with attention

c. In most instances, it is regarded as a normal waveform

d. They tend to be present anteriorly more than posteriorly

Solution. Ans-88: (c) In most instances, it is regarded as a normal waveform Ref: Ganongs Physiology, 23rd ed., p.-234 Sol :
Alpha waves generally are seen in all age groups but are most common in adults.
They occur rhythmically on both sides of the head but are often slightly higher in amplitude on the nondominant side, especially in right-
handed individuals.
A normal alpha variant is noted when a harmonic of alpha frequency occurs in the posterior head regions.
They tend to be present posteriorly more than anteriorly and are especially prominent with closed eyes and with relaxation.
Alpha activity disappears normally with attention (eg, mental arithmetic, stress, opening eyes).
In most instances, it is regarded as a normal waveform.
An abnormal exception is alpha coma, most often caused by hypoxic-ischemic encephalopathy of destructive processes in the pons (eg,
intracerebral hemorrhage).
In alpha coma, alpha waves are distributed uniformly both anteriorly and posteriorly in patients who are unresponsive to stimuli.

Correct Answer. c

(89). A 62-year-old woman eats a high carbohydrate meal. Her plasma glucose concentration rises, and this results in increased insulin
secretion from the pancreatic islet cells. The insulin response is an example of

a. Chemical equilibrium

b. End-product inhibition

c. Negative feedback

d. Positive feedback

Solution. Ans 89: (c) Negative feedback Ref: Read the text below Sol:
The increase in plasma insulin lowers the plasma glucose concentration back to normal and is an example of negative feedback.
Negative feedback opposes change and results in stability. Positive feedback would produce a further increase in plasma glucose
concentration.
Chemical equilibrium indicates a condition in which the rates of reactions in forward and backward directions are equal.
End-product inhibition occurs when the products of a chemical reaction slow the reaction (for example, by inhibiting an enzyme) that
produces them.
Feedforward control involves a command signal and does not directly sense the regulated variable (plasma glucose concentration).

Correct Answer. c

Copyright 2014 Delhi Academy of Medical Sciences, All Rights Reserved. 34/76
(90). Binding of estrogen to its steroid hormone receptor

a. Stimulates the GTPase activity of the trimeric G protein coupled to the estrogen receptor

b. Stimulates the activation of the IP3 receptor in the sarcoplasmic reticulum to increase intracellular calcium

c. Stimulates phosphorylation of tyrosine residues in the cytoplasmic domain of the receptor

d. Stimulates the movement of the hormone-receptor complex to the nucleus to cause gene activation

Solution. Ans 90: (d) Stimulates the movement of the hormone-receptor complex to the nucleus to cause gene activation Ref: Read the
text below Sol:
Steroid hormone receptors are transcriptional regulators found in the cytoplasm or in the nucleus.
These receptors are activated by the binding of steroid ligands that diffuse through lipid bilayers and enter the cytosol.
Activated steroid receptors mediate their effects by direct interaction with gene regulatory elements and do not activate G proteins or
cause binding of IP3 to the IP3-gated calcium release channel in the sarcoplasmic reticulum.
Steroid hormone receptors do not have tyrosine kinase activity and do not cause the phosphorylation of tyrosine residues in these
receptors. Steroid hormone receptors are not linked to activation of the MAP kinase pathway.
Estrogen receptors are located in the cytoplasm of cells; upon binding of estrogen, they move to the nucleus to bind to estrogen
response elements to activate gene transcription

Correct Answer. d

(91). During regulatory volume decrease, many cells will increase

a. Their volume

b. Influx of Na+

c. Efflux of K+

d. Synthesis of sorbitol

Solution. Ans 91: (c) Efflux of K+ Ref: Read the text below Sol:
K+ is the major intracellular ion; efflux of K+ will produce an osmotic flow of water out of the cell. Water exit will lead to a decrease in cell
volume.
An influx of Na+ and synthesis of sorbitol do not occur during this process because both processes would increase intracellular osmolytes
and drive water into the cell, increasing cell volume.

Correct Answer. c

(92). A decrease in sensory response while a stimulus is maintained constant is due to the phenomenon of

a. Adaptation

b. Fatigue

c. The graded response

d. Compression

Solution. Ans 92: (a) Adaptation Ref: Read the text below Sol:
When a receptor adapts, the sensation decreases although the stimulus may be unchanged.
Adaptation is largely a result of the fall in magnitude of the generator potential and is not due to fatigue.
Sensory responses are graded in response to changes in stimulus intensity regardless of the level of adaptation, and the phenomenon of
compression allows a wide range of environmental intensities to be translated into a much more narrow range of sensory responses.

Correct Answer. a

Copyright 2014 Delhi Academy of Medical Sciences, All Rights Reserved. 35/76
(93). Which type of sensory receptor provides information about the force of muscle contraction?

a. Nuclear bag fiber

b. Nuclear chain fiber

c. Golgi tendon organ

d. Bare nerve ending

Solution. Ans-93: (c) Golgi tendon organ Ref: Read the text below Sol:
The Golgi tendon organ senses the force of muscular contraction.
The nuclear chain and bag fibers, along with type Ia endings, are all components of the muscle spindle which reports muscle length and
velocity of muscle shortening.

Correct Answer. c

(94). Which cerebellar component would be abnormal in a degenerative disease that affected spinal sensory neurons?

a. Purkinje cells

b. Mossy fibers

c. Parallel fibers

d. Climbing fibers

Solution. Ans 94: (b) Mossy fibers Ref: Read the text below Sol:
Spinal input, such as from the spinocerebellar tracts, enters the cerebellum on the mossy fibers.
The climbing fibers originate from the inferior olivary nucleus of the medulla.
The other components are intrinsic to the cerebellum.

Correct Answer. b

(95). Marys law states the relationship of heart rate with?

a. Cardiac output

b. Stroke volume

c. Arterial BP

d. Presystolic volume

Solution. Ans-95: (c) Arterial BP Ref.: Read the text below Sol :
Pulse rate is inversely proportional to the blood pressure but not vice versa. This is called Marys law.

Correct Answer. c

Copyright 2014 Delhi Academy of Medical Sciences, All Rights Reserved. 36/76
(96). A fertilized ovum reaches uterine cavity in?

a. 3-4 days

b. 5-6 days

c. 7-8 days

d. 10 days

Solution. Ans-96: (a) 3-4 days Ref.: Read the text below Sol :
The fertilized ovum, beginning its division, continues on its way through the tube; its progress is quite slow. It enters the uterine cavity
3-4 days later.
The endometrium at this time is under the influence of the corpus luteum, and its glands are highly developed and actively secreting.
Implantation of the fertilized ovum occurs about 5-6 days after it has entered the uterine cavity, 7 to 10 days after ovulation.
The growth and development of the ovum before implantation are supported by materials stored in the ovum itself, the cells which cover
it, and perhaps the fluids of the uterine cavity.

Correct Answer. a

(97). The most widely used classification of mammalian peripheral nerves is Erlanger and Gasser classification, based on nerve diameter and
velocity of conduction. In this classification. In this classification, proprioceptory fibers belong to which group?

a. Aalpha

b. Abeta

c. Agamma

d. Adelta

Solution. Ans-97: (a) Aalpha Ref.: Read the text below Sol :
Fibre Type Function

A alpha Proprioception, Somatic motor

A beta Touch, Pressure, Motor

Agamma Motor to muscle spindles

Adelta Pain, Cold, Touch

B Preganglionic autonomic

C Dorsal root Pain, temperature, mechanoception, reflex responses

C Sympathetic Post ganlionic sympathetic.

Correct Answer. a

Copyright 2014 Delhi Academy of Medical Sciences, All Rights Reserved. 37/76
(98). Which of the following causes shift of oxygen dissociation curve to the right?

a. Decreased pH

b. Decreased PCO2

c. Decreased temperature

d. Decreased 2, 3-BPG

Solution. Ans-98: (a) Decreased pH Ref.: Read the text below Sol :
Factors causing the shift of the Oxygen Dissociation Curve are : (very important)
Left Shift Right Shift

Alkalosis or increased pH Acidosis or decreased pH

Decreased PCO2 Increased PCO2

Decreased temperature Increased temperature

Decreased 2, 3-BPG Increased 2, 3 BPG

Correct Answer. a

(99). Which factor deficiency is known as Hemophilia C?

a. 8

b. 9

c. 10

d. 11

Solution. Ans-99: (d) 11 Ref.: Read the text below Sol : HEMOPHILIA C
Haemophilia C (also known as plasma thromboplastin antecedent (PTA) deficiency or Rosenthal syndrome) is a mild form of haemophilia
affecting both sexes. However, it predominantly occurs in Jews of Ashkenazi descent. It is the fourth most common coagulation disorder
after von Willebrand's disease and haemophilia A and B.
It is caused by a deficiency of coagulation factor XI and is distinguished from haemophilia A and B by the fact it does not lead to
bleeding into the joints. Furthermore, it has autosomal inheritance, since the gene for factor XI is located on chromosome 4 (close to the
prekallikrein gene); and it is not completely recessive, individuals who are heterozygous also show increased bleeding. Many mutations
exist, and the bleeding risk is not always influenced by the severity of the deficiency. Treatment is usually not necessary, except in
relation to operations, leading to many of those having the condition not being aware of it. In these cases, fresh frozen plasma or
recombinant factor XI may be used, but only if necessary.

Correct Answer. d

(100). The average cerebral blood flow in young adults is:

a. 35 ml/100gm/min

b. 55 ml/100gm/min

c. 65 ml/100gm/min

d. 75 ml/100gm/min

Solution. Ans-100: (b) 55 ml/100gm/min Ref.: Read the text below Sol :
The average cerebral blood flow in young adults is 54ml/100gm/min. average adult brain weighs about 1400 gms, so the flow for the
whole brain is about 756ml/min (15% of cardiac output).

Correct Answer. b

Copyright 2014 Delhi Academy of Medical Sciences, All Rights Reserved. 38/76
(101). Which among the following is true regarding oxytocin?

a. Synthetic peptide of 8 amino acids

b. Stimulates PG synthesis

c. In humans, high doses can result in hypernatremia

d. Responsible for contraction of uterus during first trimester.

Solution. Ans-101: (b) Stimulates PG synthesis Ref.: Read the text below Sol : Oxytocin stimulates the synthesis of prostaglandins by
activating phospholipase enzyme. OXYTOCIN
Oxytocin is a nonapeptide (9 amino acids) synthesized in Paraventricular nuclei of hypothalamus, lossely bound to neurophysin-I and
relased in the nerve endings (Herring bodies) of the hypothalamo-hypophyseal tract fibres in the posterior pituitary and finally released
as free form in circulation.
Mechanism of action : it triggers increase in intracellular Ca2+ levels it stimulates PG synthesis by activating phospholipases
Action : it is responsible for milk ejection reflex (neuroendocrine reflex), uterine musculature contraction during labour (neuroendocrine
reflex), and sperm transport along the female genital tract.
Factors influencing secretion :
Throughout labour, large quantities of oxytocin is released by means of positive feedback mechanism.
In lactating women, genital stimulation and emotional stimuli also enhance oxytocin. The secretion of oxytocin is increased by stressful
stimuli & is inhibited by alcohol.
In high doses, it can result in water intoxication and natriuresis in several species.
Vasopressin (nonapeptide) is synthesized in Supraoptic nuclei of hypothalamus.

Correct Answer. b

(102). Which is the earliest site of erythropoiesis in a developing foetus?

a. Yolk sac

b. Liver

c. Bone marrow

d. Spleen

Solution. Ans-102: (a) Yolk sac Ref.: Read the text below Sol : Sites of erythropoiesis
Age Organ
rd
3 weeks 3 month of gestation Mesodermal wall of yolk sac

rd th
3 month 7 month of gestation Liver (main), spleen, lymph nodes

Remaining months of gestation & after birth Red marrow

Birth 5 years All flat & long bones

5 years 20 years More in flat & lesser in long bones


20 years Predominantly in flat bones like vertebra,

sternum, ribs & ilia

Correct Answer. a

Copyright 2014 Delhi Academy of Medical Sciences, All Rights Reserved. 39/76
(103). The only excitatory output of basal ganglia comes from :

a. Putamen

b. Subthalamic nucleus

c. Globus pallidus

d. Substantia nigra

Solution. Ans-103: (b) Subthalamic nucleus Ref.: Read the text below Sol :
Two major excitatory inputs (glutamate) to the basal ganglia are from cerebral cortex (corticostriate pathway) & intralaminar nuclei of
thalamus (thalamostriate pathyway) & both terminate in the Striatum
Only excitatory output of basal ganglia is from the sub-thalamic nucleus (glutamate)

Correct Answer. b

(104). The last step in the biosynthesis of vit D takes place in:

a. Skin

b. Liver

c. Kidney

d. Intestine

Solution. Ans-104: (c) Kidney Ref.: Read the text below Sol :

Correct Answer. c

Copyright 2014 Delhi Academy of Medical Sciences, All Rights Reserved. 40/76
(105). All of the following hormones increase erythropoietin secretion except:

a. Thyroxine

b. Growth hormone

c. Glucocorticoids

d. Oestrogen

Solution. Ans-105: (d) Oestrogen Ref.: Read the text below Sol : Factors Increasing Erythropoietin Secretion
Hypoxia (Most potent stimulus)
High altitudes
Alkalosis
Anemia either due to blood loss or any other condition stimulates EPO secretion
Vit B12 & Folate : DNA synthesis
Iron : Heme synthesis
Hormones : Thyroxine, Growth hormone, prolactin, ACTH, Glucocorticoids, Androgens, Catecholamines, PGs stimulate erythropoietin
secretion.
Hemolysates (products released from the RBC destruction)
Nucleotides : cAMP, NAD, NADP
Vasoconstrictor drugs renal hypoxia stimulate EPO secretion
Factors Decreasing Erythropoietin Secretion
Adenosine antagonists Eg. Theophylline
Oestrogen decreases EPO secretion by decreasing the globin synthesis in the liver.

Correct Answer. d

(106). Which of the following sets of attributes best characterizes cardiac muscle?

a. Large cells, electrically isolated, neurally stimulated

b. Small cells, electrically coupled,chemically stimulated

c. Small cells, electrically coupled, spontaneously active

d. Small cells, electrically isolated, spontaneously active

Solution. Ans 106: (c) Small cells, electrically coupled, spontaneously active Ref: Read the text below Sol:
Cardiac muscle has small cells that must be coupled electrically for communication to occur.
Because it receives no motor innervation, it must be spontaneously active.

Correct Answer. c

(107). Vomiting centre is situated in the :

a. Hypothalamus

b. Midbrain

c. Pons

d. Medulla

Solution. Ans-107: (d) Medulla Ref: Ganongs Physiology, 23rd ed., p.-474 Sol :
Vomiting is believed to be controlled by two distinct brain centres-the vomiting centre and the chemoreceptor trigger zone-both located
in the medulla oblongata.
The vomiting centre initiates and controls the act of emesis, which involves a series of contractions of the smooth muscles lining the
digestive tract.

Correct Answer. d

Copyright 2014 Delhi Academy of Medical Sciences, All Rights Reserved. 41/76
(108). Optic nerve is :

a. 1st order neuron

b. 2nd order neuron

c. 3rd order neuron

d. 4th order neuron

Solution. Ans-108: (b) 2nd order neuron Ref: Read the text below Sol :
In the case of the optic nerve and tracts there are three separate neurons linked together, extending from the retina to the occipital
cortex.
The sensory end-organ consists of the rods and cones.
The first neuron begins in the outer plexiform or granular layer, and ends in the inner plexiform or granular layer. The cell-body
governing the nutrition of this fibre is the bipolar cell, which cells forms the inner nuclear layer.
This neuron, although extremely short is the true optic nerve in the physiological sense, and corresponds to the long fibre with runs from
the sole of the foot up into the spinal cord.
The second neuron in the retina begins in the inner nuclear layer and runs from here up to the primary optic ganglia, i.e. chiefly the
external geniculate body.
This is the anatomical optic nerve.
The cell governing the nutrition of this fibre is the ganglion cell.

Correct Answer. b

(109). Less O2 saturation in blood is seen in :

a. R-L shunt

b. LV obstruction

c. RV obstruction

d. L-V shunt

Solution. Ans-109: (a) R-L shunt Ref: Ganongs Physiology, 23rd ed., p.-620 Sol :
Shunting of blood from the right side to the left side of the circulation (right-to-left shunt) is powerful cause of hypoxemia.
The shunt may be intracardiac or may be intrapulmonary.
It has been traditionally thought that this cause could be readily distinguished from the others as the only cause that cannot be
corrected by the administration of 100% oxygen.

Correct Answer. a

(110). Which is responsible for respiratory drive:

a. O2

b. CO

c. CO2

d. Bicarbonate ions

Solution. Ans-110: (c) CO2 Ref: Ganongs Physiology, 23rd ed., p.-516 Sol :
Although the body requires oxygen for metabolism, low oxygen levels do not stimulate breathing. Rather, respiratory centre is directly
stimulated by higher carbon dioxide levels or excess hydrogen ions in the blood.
As a result, breathing low-pressure air or a gas mixture with no oxygen at all (such as pure nitrogen) can lead to loss of consciousness
without ever experiencing air hunger.
The respiratory centers try to maintain an arterial CO2 pressure of 40 mm Hg. With intentional hyperventilation, the CO2 content of
arterial blood may be lowered to 10-20 mm Hg (the oxygen content of the blood is little affected), and the respiratory drive is diminished.

Correct Answer. c

Copyright 2014 Delhi Academy of Medical Sciences, All Rights Reserved. 42/76
(111). On decreasing the alveolar ventilation to half of the normal, what will be the change in PaCO2?

a. PaCO2 doubles immediately

b. PaCO2 halves immediately

c. PaCO2 doubles over next several minutes

d. PaCO2 halves over next several minutes

Solution. Ans:111: (c)PaCO2doubles over next several minutes Ref: Ganong 22nd/659; Sol: Alveolar ventilation
Alveolar ventilation & PaCO2 are inversely related.
Alveolar ventilation is a key determinant of PaCO2
If alveolar ventilation is halved, PaCO2-will double or vice versa.

Correct Answer. c

(112). Gastric secretion of H+ion:

a. Is by chief cells

b. Is regulated in part by number of H+ pumps

c. Is enhanced by carbonic anhydrase inhibitors

d. Occurs in exchange for luminal Na+ion

Solution. Ans:112: (b) Is regulated in part by number of H+ pumps Ref: Ganong 22nd/493 Sol: Hydrochloric Acid Secretion
Secreted by parietal cells, not by the chief cells.
H+-k+ ATPase in the aplical membrance of the parietal cells that pumps H+ against against a concentration gradient.
TheH+ that is exdruded comes from H2CO3 , and H2CO3 in turn is formed by the hydration ofCO2.
This latter reaction is catalyzed by carbonic anhydrase, and the parietal cells are particularly rich in this enzyme.
Carbonic anhydrase inhibitors decreases HCI secretion.
Gastric secretin of H+ ions is reghulated in part by number of H+ pumps

Correct Answer. b

(113). Platelets contribute to clot retraction by the following mechanisms except:

a. By release of fibrin stabilizing factor

b. By activation of thrombosthenin, actin and myosin

c. By release of Ca+ions form its mitochondria

d. By activation of factor XIII

Solution. Ans:113: (d) By activation of factor XIII Ref: Ganong 22nd/542 Sol:
Clot strength (not the clot retraction) derives in part from factor XIIIa-mediated cross linking, and because 50% of factor XIII are
platelet derived, platelet, factor XIII and clot strength all correlate with each other. Clot retraction doesnt depend on factor XIII.

Correct Answer. d

Copyright 2014 Delhi Academy of Medical Sciences, All Rights Reserved. 43/76
(114). The first reflex to return after recovery from spinal shock is:

a. Stretch reflex

b. Flexor reflex

c. Stepping reflex

d. Postural antigravity reflex

Solution. Ans:114: (b) Flexor reflex Ref: Ganong 22and/207-208 Sol: Spinal shock
In all vertebrates, transaction of the spinal cord is followed by a period of spinal shock during which all spinal reflex response are
depressed
The duration of spinal shock is proportionate to the degree of encephalization of motor function in various species, varying from a few
minutes in frogs and rats to a minimum of two weeks in humans.
The cause of spinal shock is uncertain.
The first reflex response to reappear after spinal shock wears off is the flexor response i.e. a slight contraction of the leg flexors and
adductor in response to a painful stimulus.

Correct Answer. b

(115). At the end of normal expiration:

a. The outward recoil tendency of the chest

b. The volume inside the lung is expiratory reserve volume

c. The chest wall has a tendency to move inwards

d. Alveolar pressure is usually negative.

Solution. Ans: 115: (a) The outward recoil tendency of the chest Ref: Ganong 22nd/651 Sol: Expiration
At the end of quiet expiration, the tendency of the lungs to recoil from the chest wall is just balanced by the tendency of the chest wall to
recoil in the opposite direction
The intrapulmonary pressure is zero at a lung volume that corresponds to the volume of gas in the lungs at the end of normal quiet
expiration this is relaxation volume, which equals the functional residual capacity

Correct Answer. a

(116). The mechanism involved in the absorption of glucose from the small intestine is:

a. Passive diffusion

b. Facilitated diffusion

c. Secondary active contransport with sodium

d. Actively by insulin dependent uptake

Solution. Ans:116: (c) Secondary active contransport with sodium Ref: Ganong 22nd/33 Sol:
The mechanism involved in the absorption of glucose from the small intestine is secondary active cotransport with sodium.

Correct Answer. c

Copyright 2014 Delhi Academy of Medical Sciences, All Rights Reserved. 44/76
(117). Stimulation of proximal cut end of vagus causes:

a. Apnea

b. Decreased heart rate

c. Decreased BP

d. +BP

Solution. Ans:117: (a) Apnea Ref: Ganong 22nd/672 Sol:


Stretching of lungs during inspiration initiates impulses in afferent pulmonary vagal fibres.
These impulses inhibit inspiratory discharges
Stimulation of the proximal cut end of vagus therefore inhibit inspiration and produce apnoea.
Effect of stimulation of proximal cut end of vagus on blood pressure and heart rate not cons

Correct Answer. a

(118). Which one of the following is released by blood platelets during haemorrhage to produce vasoconstriction?

a. Serotonin

b. Histamine

c. Thrombosthenin

d. Bradykinin

Solution. Ans:118: (a) Serotonin Ref: Ganong 22nd/542 Sol:

Correct Answer. a

(119). Decreasing the resistance of the afferent arteriole in the glomerulus of the kidney will decrease

a. The renal plasma flow

b. The filtration fraction

c. The glomerular filtration rate

d. None of the above

Solution. Ans-119: (d) None of the above Ref: Read the text below Sol:
Renal plasma flow, filtration fraction, the oncotic pressure, and the filtration rate all increase when the afferent arteriolar resistance is
decreased.
The renal blood flow increases because total renal resistance is less.
Decreasing renal resistance also increases the glomerular capillary pressure, which results in an increase in filtration fraction.
Because more fluid is filtered out of the glomerular capillaries and no plasma protein is removed, the oncotic pressure rises.
The glomerular filtration rate (GFR) is proportional to the glomerular capillary pressure and the renal plasma flow (RPF). Since both of
these increase, so does the GFR.

Correct Answer. d

Copyright 2014 Delhi Academy of Medical Sciences, All Rights Reserved. 45/76
(120). Which one of the following statements about aldosterone is correct?

a. It produces its effect by activating cAMP

b. It produces its effect by increasing distal tubular permeability to sodium

c. It causes an increased reabsorption of hydrogen ion

d. It has its main effect on the proximal tubule

Solution. Ans-120: (b) It produces its effect by increasing distal tubular permeability to sodium Ref: Read the text below Sol:
Aldosterone binds to an intracellular receptor that causes an increased synthesis of a variety of proteins, including K+ and Na+ ion
channels and Na+-K+-ATPase, which together act to increase Na+ reabsorption and K+ secretion by the tubular cells of the distal
nephron.
H+ secretion is also enhanced by aldosterone.
Aldosterone secretion is stimulated by a decrease in blood volume (through the renin-angiotensin system) and by increased plasma K+
concentrations.
High blood pressure, if it has any effect on aldosterone, will cause a decrease in its secretion.

Correct Answer. b

(121). The syndrome of inappropriate antidiuretic hormone secretion (SIADH) is caused by the excess release of ADH. SIADH will cause an
increase in

a. Concentration of plasma sodium

b. Intracellular volume

c. Urinary flow

d. Plasma oncotic pressure

Solution. Ans-121: (b) Intracellular volume Ref: Read the text below Sol:
The increased secretion of antidiuretic hormone (ADH) increases the permeability of the distal nephron to water and therefore increases
the reabsorption of water from the kidney.
The excessive reabsorption of water dilutes the extracellular fluid, producing a decrease in plasma sodium, osmolarity, and oncotic
pressure.
The decreased extracellular osmolarity causes water to flow from the extracellular fluid compartment into the intracellular fluid
compartment, increasing intracellular volume.
Because more water is being reabsorbed, less is excreted and urinary flow is decreased

Correct Answer. b

(122). Which one of the following comparisons between the distal nephron and the proximal tubule is correct?

a. The distal nephron is more permeable to hydrogen ion than the proximal tubule

b. The distal nephron is less responsive to aldosterone than the proximal tubule tubule

c. The distal nephron has a more negative intraluminal potential than the proximal tubule

d. The distal nephron secretes less potassium than does the proximal tubule

Solution. Ans-122: (c) The distal nephron has a more negative intraluminal potential than the proximal tubule Ref: Read the text below
Sol:
The distal nephron has a negative luminal potential because it is poorly permeable to negatively charged ions.
Therefore, when Na+ is reabsorbed, negatively charged ions, primarily Cl, lag behind, producing a negative intraluminal potential.
Although a similar situation occurs in the proximal tubule, the proximal tubule has a higher permeability to Cl and, therefore, does not
develop as large a negative intraluminal potential.
The distal nephron is less permeable to hydrogen than the proximal tubule. Aldosterone increases Na+ reabsorption from the distal
nephron but has no effect on the proximal tubule. K+ is reabsorbed from the proximal tubule and secreted by the distal nephron.
Although the amount of H+ excreted each day is determined by the amount of H+ secreted into the distal nephron, the proximal tubule
secretes much more H+ than the distal nephron.
However, almost all of the H+ secreted in the proximal tubule is reabsorbed in associationwith the reabsorption of HCO3.

Correct Answer. c

Copyright 2014 Delhi Academy of Medical Sciences, All Rights Reserved. 46/76
(123). Severe head trauma that results in interruption of the hypothalamo-hypophyseal portal vessels would most likely lead to which of the
following conditions?

a. Hyperpigmentation

b. Acromegaly

c. Insulin resistance

d. Hyperprolactinemia

Solution. Ans-123: (d) Hyperprolactinemia Ref: Read the text below Sol:
Prolactin is the only anterior pituitary hormone that is under tonic inhibitory control by hypothalamic, hypophysiotropic hormones.
(Dopamine is the prolactin-inhibiting hormone.) Interruption of hypophyseal portal vessels leads to increased prolactin secretion and
hyperprolactinemia.
Hyperpigmentation is associated with increased plasma levels of adrenocorticotropic hormone (ACTH)
Interruption of the hypophyseal portal vessels would result in decreased secretion of ACTH because corticotropin-releasing hormone
would not be carried to the anterior pituitary.
Both growth hormone and cortisol have peripheral "anti-insulin" effects and may produce insulin resistance. Interruption of the
hypophyseal portal vessels would decrease ACTH (and thus cortisol) and growth hormone. Hence, insulin sensitivity would be, if any-
thing, increased.

Correct Answer. d

(124). Which statement about erythropoietin is not correct ?

a. Synthesis is facilitated by hypoxia

b. Synthesis occurs predominantly in the kidneys

c. Synthesis is facilitated by moving to a high altitude

d. Renal tubular cells secrete erythropoietin

Solution. Ans-124: (d) Renal tubular cells secrete erythropoietin Ref: Read the text below Sol:
Erythropoietin is a glycoprotein that is secreted into the blood stream by endothelial cells of peritubular capillaries in the renal cortex.
Erythropoietin enhances the production of red blood cells. Loss of renal mass can therefore lead to anemia.
Approximately 85% of erythropoietin is synthesized in the kidneys, and an additional 15% is produced by the liver.
An abrupt move to a high altitude causes a decrease in PO2 levels and therefore hypoxia. As a result, erythropoietin synthesis is
stimulated.
Erythropoietin has a half-life in the circulation of about 5 hours. The predominant site of synthesis is the kidneys, whereas the
predominant site of inactivation is the liver.

Correct Answer. d

(125). Inhibitory postsynaptic potentials (IPSPs) commonly occur at which of the following sites ?

a. The anterior gray horn of the cord, dorsal root ganglia, and muscle endplates

b. The anterior gray horn of the cord and dorsal root ganglia, but not at muscle endplates

c. The anterior gray horn of the cord, but not at dorsal root ganglia or muscle endplates

d. The anterior gray horn of the cord and muscle endplates, but not at dorsal root ganglia

Solution. Ans-125: (c) The anterior gray horn of the cord, but not at dorsal root ganglia or muscle endplates Ref: Read the text below
Sol:
Control of skeletal muscle stimulation occurs at the anterior gray horn in the spinal cord where numerous excitatory postsynaptic
potentials (EPSPs) and inhibitory postsynaptic potentials (IPSPs) occur on the alpha motor neurons.
The motor neurons cannot inhibit at the neuromuscular junction; they only excite through
endplate potentials.
There are no synapses at the dorsal root ganglion, just the cell bodies of sensory neurons.

Correct Answer. c

Copyright 2014 Delhi Academy of Medical Sciences, All Rights Reserved. 47/76
(126). In the eye, the physiological reflex of papillary constriction during accommodation involves all of the following structures except :-

a. The optic nerve

b. The medial geniculate body

c. The cerebral cortex

d. The Edinger-Westphal nucleus

Solution. Ans-126: (b) The medial geniculate body Ref: Read the text below Sol:
This reflex involves all the structures except the medial geniculate body of the thalamus, which is concerned with the auditory pathway.
The lateral geniculate body is involved with the eye and is part is this pupillary reflex.

Correct Answer. b

(127). Systolic BP is highest at which site :-

a. Aorta

b. Renal artery

c. Right atrium

d. Pulmonary artery

Solution. Ans-127: (b) Renal artery Ref: Read the text below Sol:
Pressure in arterial sides are always higher. Systolic BP is more in downstream arteries than aorta because of reflection of pressure
waves at branch points.
The pressure in any vessel below heart level is increased and that in any vessel above heart level a decreased by the effect of gravity.

Correct Answer. b

(128). True statements about the kidney include:

a. The blood flow accounts for 75% of the total cardiac output.

b. The blood flow is greater in the renal medulla than in the cortex.

c. Over 99% of filtered water is reabsorbed by the kidney.

d. Antidiuretic hormone decreases the water permeability of the collecting duct.

Solution. Ans-128: (c) Over 99% of filtered water is reabsorbed by the kidney Ref: Read the text below Sol:
The kidneys receive about 1.2 liters of blood per minute which is about 25% of the cardiac output
The blood flow is 20 times greater in the renal cortex than the renal medulla. Over 99% of the filtered water is reabsorbed by the kidney.
Antidiuretic hormone increases the water permeability of the collecting duct and hence increases water retention.
Kidney secrets erythropoietin which is essential for red blood cell production.

Correct Answer. c

Copyright 2014 Delhi Academy of Medical Sciences, All Rights Reserved. 48/76
(129). Which of the following has serous secretion?

a. Pancreas

b. parotid gland

c. Sub mandibular salivary gland

d. Minor salivary gland

Solution. Ans-129: (b) parotid gland Ref.: Read the text below Sol :
Parotid gland has purely serous acini
Sub-mandibular and sub-lingual have both serous and mucinous scini
Minor salivary glands are predominantly mucinous

Correct Answer. b

(130). The most important cholorectic is

a. Bile salt

b. CCK

c. Secretin

d. Gastrin

Solution. Ans-130: (a) Bile salt Ref: Read the text below Sol:
. Cholagogue is the one which causes gall bladder contraction, thereby increasing flow (eg) CCK PZ
. Cholorectic increases secretion of bile(eg) Bile salt

Correct Answer. a

(131). Regarding cytoskeleton true is,

a. Microtubles and microfilaments are the two types of cytoskeleton

b. Microtubules are are the components of mitotic spindle.

c. Centriole has a central doublet surrounded by nine doublets of tubulin

d. F actin polymerises to form G actin

Solution. Ans-131: (b) Microtubules are are the components of mitotic spindle Ref: Read the text below Sol: Cytoskeleton is of three
types: a. Microtubules :
25nm in diameter
They are of two types 1 tubulin
They are the components of mitotic spindle
They take part in intracellular transportation
Microbule based molecular motor includes
b. Intermediate filament : Keratin,Vimentin,Neurofilament c. Microfilament :
Non muscle actins , polymerise to form F actin
Microfilament based molecular motor:- Myosin I, Myosin II

Correct Answer. b

Copyright 2014 Delhi Academy of Medical Sciences, All Rights Reserved. 49/76
(132). Synaptic transmission between pain fibers from the skin and spinal cord neurons is mediated by

a. Acetylcholine

b. Substance P

c. Endorphins

d. Somatostatin

Solution. Ans-132: (b) Substance P Ref: Read the text below Sol :
Substance P is an 11-amino acid polypeptide found in neurons within the hypothalamus and spinal cord.
It is released from small A delta and C fibers that relay information from nociceptors to neurons within the substantia gelatinosa of the
spinal cord.
Endorphins and other opioid neurotransmitters may partially inhibit the perception of pain by presynaptically inhibiting the release of
substance P from nociceptor afferent fibers.

Correct Answer. b

(133). Which of the following would not occur following a major blood loss?

a. Vagal tone to the SA node would be decreased which would then increase the heart rate.

b. An increase in sympathetic output in arterioles would increase the total peripheral resistance, except in the brain and heart.

c. Stroke volume would increase while heart rate decreased. Together this would increase the cardiac output.

d. Sympathetic output to the veins would increase, resulting in increased venous pressure, venous return, and cardiac output.

Solution. Ans-133: (c) Stroke volume would increase while heart rate decreased. Together this would increase the cardiac output. Ref:
Read the text below Sol:
. A sudden drop in blood volume (eg. due to severe blood loss) results in low venous return and therefore decreased stroke volume.
. Sympathetic activity increases heart rate, maintaining cardiac output.

Correct Answer. c

(134). All are true about Growth Hormone except

a. Actions mediated by somatomedins produced in liver

b. Causes decreased glucose uptake in skeletal muscle and fat

c. Involved in the conversion of chondrocytes into osteogenic cells

d. Prevents the mobilization of fatty acids from adipose tissue

Solution. Ans-134: (d) Prevents the mobilization of fatty acids from adipose tissue Ref.: Read the text below Sol :
Growth hormone facilitates the mobilization of free fatty acids from the adipose tissue and these are used to supply most of the energy
for the bodys cells, thus acting as a potent protein sparer.
GROWTH HORMONE :
Growth hormone, also known as somatotropic hormone or somatotropin, is a small protein molecule secreted by somatotropes of
anterior pituitary that contains 191 AA in a single chain with molecular weight 22, 005
It exerts most of its effects through growth factors called somatomedins (earlier called as Sulfation factor) secreted by the lvier and
other tissues. GH acts through JAK-STAT Kinase
The principal circulating somatomedins are insulin-like growth factor I (IGF-I, somatomedin C) & IGF-II
The basal plasma GH level in adult humans is 3mg/ml and its half-life is 6-20 mins
Site of inactivation : Liver & kindey

Correct Answer. d

Copyright 2014 Delhi Academy of Medical Sciences, All Rights Reserved. 50/76
(135). Gap junctions are intercellular connection that serve :

a. To keep cells separated & allow for transport across a tissue barrier

b. As a regulated cytoplasmic bridge for sharing of small molecules between cells

c. As a barrier to prevent protein movement within the cell membrane

d. For constitutive exocytosis that occurs between adjacent cells

Solution. Ans-135: (b) As a regulated cytoplasmic bridge for sharing of small molecules between cells Ref.: Read the text below Sol :
Gap junctions are intercellular connections that form a cytoplasmic Tunnel which permits the transfer of ions, gugars, amino acids &
other solutes with mol, weight < 1000 Daltons to pass between the cells without entering the ECF.

Correct Answer. b

(136). All are true about baroreceptors except :

a. its stimulation produces bradycardia & hypotension

b. Afferent nerves from IX & X cranial nerves are called buffer nerves

c. Is active only during systole

d. Most of the afferents end in the nucleus of tractus solitarius.

Solution. Ans-136: (c) Is active only during systole Ref.: Read the text below Sol :
Baroreceptors are stimulated within the range of 80-220 mmHg diastolic BP and MAP 50-200 mmHg.
They are more sensitive to pulsatile pressure than to constant pressure.
At a lower MAP, the overall firing rate is considerably reduced with activity only occurring during systole.
At normal BP levels (about 100mmHg MAP), a burst of action potentials appears in a single baroreceptor fiber during systole, but there
are few action potentials in early diastole also.
The threshold for eliciting activity in the carotid sinus nerve is 50 mmHg; maximal activity at 200mmHg, with activity throughout the
cardiac cycle (both systole & diastole).
In short, the rate of discharge from baroreceptors is directly proportional to the MAP (arterial blood pressure)

Correct Answer. c

(137). Which among the following is a cardio inhibitory centre?

a. RVLM

b. CVLM

c. Nucleus ambiguus

d. NTS

Solution. Ans-137: (c) Nucleus ambiguus Ref.: Read the text below Sol :
Major sources of excitatory input to sympathetic nerves controlling the vasculature are a group of neurons located near the pial surface
of the medulla in the rostral ventrolateral medulla (RVLM).
This region is sometimes called Vasomotor area.
Stimulation of this area produces vasoconstriction & increase in BP, C.O. and stroke volume.
During sleep, the activity of VMC is decreased gut not absent.
Cardiac inhibitory centre : Nucleus ambiguus and dorsal motor nucleus of vagus in the medulla. They receive excitatory inputs from NTS
and inhibit the heart via the vagus nerve. So, referred to as Cardiac.

Correct Answer. c

Copyright 2014 Delhi Academy of Medical Sciences, All Rights Reserved. 51/76
(138). Which of the following phases contributes the maximum to gastric acid secretion?

a. Cephalic

b. Gastric

c. Intestinal

d. Basal

Solution. Ans-138: (b) Gastric Ref.: Read the text below Sol : Cephalic phase
The cephalic phase occurs even before the food enters the stomach.
Results from sight, smell, though or taste of dood.
Neurogenic signals originate in the cerebral cortex and in the appetite centres of amygdale and hypothalamus which are transmitted
through the dorsal motor nuclei of vagus and thence through the vagus nerves to the stomach
Accounts for 20% of the gastric acid secretion.
Gastric phase
Once the food enters the stomach, it excites :
Long vagovagal reflexes from the stomach to the brain & back to the stomach.
Local enteric reflexes
The gastrin mechanism
Accounts for 70% of the gastric acid secretion (about 1500 ml)

Correct Answer. b

(139). The number of T tubules per sarcomere in a skeletal muscle is:

a. 1

b. 2

c. 3

d. 4

Solution. Ans-139: (b) 2 Ref.: Read the text below Sol : Sarcotubular system
T-tubules are actually internal extensions of the sarcolemma which penetrate all the way from one side of the muscle fibre to the
opposite side and open to the exterior (ECF).
They are responsible for rapid transmission of action potential from sarcolemma to the deep interior of the muscle fibre.
T-tubules Skeletal muscle : present at A-1 junction (i.e.- two per sarcomere)
Smooth muscle : absent
Cardiac muscle : present at Z-line
Sarcoplasmic reticulum : composed of two parts
Large chambers called terminal cisternae that about the T-tubules on each side. They are collectively called a triad.
Longitudinal tubules (L-tubules) that surround all surfaces of the actual contracting myofibrils. They correspond to the endoplasmic
reticulum of other cells.

Correct Answer. b

Copyright 2014 Delhi Academy of Medical Sciences, All Rights Reserved. 52/76
(140). Turbulent flow is likely to occur if:

a. Density of fluid decreases

b. Diameter of the vessel is doubled

c. Velocity is decreased

d. Viscosity is increased

Solution. Ans-140: (a) Density of fluid decreases Ref.: Read the text below Sol : Reynolds Number
The tendency for turbulent flow increases in direct proportion to the velocity of blood flow, the diameter of the blood vessel, and the
density of the blood, and is inversely proportional to the viscosity of the blood, in accordance with the following equation.
Re = DV/
Where, Re is the Reynolds number (the measure of the tendency for turbulence to occur)
V is the velocity of the flow (cm/s)
D is the diameter of the vessel (cm) is the density of the fluid
is the viscosity of the fluid (poise)

Correct Answer. a

(141). What happens to a neuron in hyperkalemia?

a. The neuron becomes more excitable

b. The neuron becomes less excitable

c. RMP of the neuron becomes more negative

d. The nerve becomes hyperpolarized

Solution. Ans-141: (a) The neuron becomes more excitable Ref.: Read the text below Sol :
When the cell membrane is permeable to more than one ion, then the cell potential is calculated by Goldman-Hodgkin-katz equartion
If the extracellular level of K+ is increased (Hyperkalemia), the resting potential moves closer to the threshold (K+ effux of K+ ions move
from ECF to ICF, adding more positivity to RMP- depolarization) for eliciting an action potential, thus neuron becomes more excitable.

Correct Answer. a

Copyright 2014 Delhi Academy of Medical Sciences, All Rights Reserved. 53/76
(142). 95% of the alveolar surface is lined by:

a. Type 1 alveolar cells

b. Type 2 alveolar cells

c. Pseudostratified columnar

d. Keratinised squamous

Solution. Ans-142: (a) Type 1 alveolar cells Ref.: Read the text below Sol :

Surfactants :
Surfactant is a surface acting agent that lines the epithelium of the alveoli in the lungs and decreases the surface tension in the alveolar
membrane.
Two types of epithelial cells lining the alveoli are. Type I cells
Flat cells
Primary lining cells of the alveoli covering 95% of the alveolar epithelial surface area.
Type II cells (granular pneumocytes)
Are thicker and contain numerous lamellar bodies
Cover only 5% of the surface area.
Important in alveolar repair & production of surfactant

Correct Answer. a

(143). Which is the type of hypoxia that does not produce chemoreceptor stimulation?

a. Hypoxic hypoxia

b. Anaemic hypoxia

c. Ischemic hypoxia

d. Stagnant hypoxia

Solution. Ans-143: (b) Anaemic hypoxia Ref.: Read the text below Sol :
Stimuli for peripheral chemoreceptors : Low PaO2, high PCO2, low PH.
In anemic hypoxia, PaO2 is normal, hence no chemoreceptor stimulation occurs.
Other three types of hypoxia stimulate chemoreceptors in one way or other.

Correct Answer. b

Copyright 2014 Delhi Academy of Medical Sciences, All Rights Reserved. 54/76
(144). In the presence of a drug that blocks all effects of norepinephrine and epinephrine on the heart, the autonomic nervous system can

a. Raise the heart rate above its intrinsic rate

b. Lower the heart rate below its intrinsic rate

c. Raise and lower the heart rate above and below its intrinsic rate

d. Neither raise nor lower the heart rate from its intrinsic rate

Solution. Ans-144: (b) Lower the heart rate below its intrinsic rate Ref: Read the text below Sol:
Activation of parasympathetic nerves to the heart would lower the heart rate below its intrinsic rate.
However, with all effects of norepinephrine and epinephrine blocked, the sympathetic nervous system cannot raise the heart rate above
its intrinsic rate.
The withdrawal of parasympathetic nerve tone could only raise the heart rate to the intrinsic rate.

Correct Answer. b

(145). Parasympathetic stimulation induces salivary acinar cells to release the protease

a. Bradykinin

b. Kallikrein

c. Kininogen

d. Kinin

Solution. Ans-145: (b) Kallikrein Ref: Read the text below Sol:
Parasympathetic stimulation induces the release of kallikrein by the salivary acinar cells, which converts kininogen to form lysyl-
bradykinin (a potent vasodilator).
Bradykinin is a vasoactive peptide.
Kininogen is the precursor for kinins.
Kinins include bradykinin and lysyl-bradykinin.
Aminopeptidase releases amino acids from the amino end of peptides and is found in the brush border membrane and cytoplasm of
enterocytes.

Correct Answer. b

(146). Which protein is absent in saliva?

a. Lactoferrin

b. Amylase

c. Mucin

d. Intrinsic factor

Solution. Ans-146: (d) Intrinsic factor Ref: Read the text below Sol:
Intrinsic factor is secreted by the parietal cells of the stomach and is not secreted by the salivary glands.
Lactoferrin, amylase, mucin, and muramidase are found in saliva.

Correct Answer. d

Copyright 2014 Delhi Academy of Medical Sciences, All Rights Reserved. 55/76
(147). Unlike other GI secretions, salivary secretion is controlled almost exclusively by the nervous system and is significantly inhibited by

a. Atropine

b. Pilocarpine

c. Cimetidine

d. Aspirin

Solution. Ans-147: (a) Atropine Ref: Read the text below Sol:
Salivary secretion is inhibited by atropine.Atropine is an anticholinergic drug that competitively inhibits ACh at postganglionic sites,
inhibiting parasympathetic activity.
Pilocarpine actually stimulates salivation because of its muscarinic action.Cimetidine is an antagonist for the histamine H2 receptor.
Aspirin is the most widely used analgesic (pain reducer), antipyretic (fever reducer), and anti-inflammatory drug.
Omeprazole inhibits the H+/K+ATPase and, thus, inhibits acid secretion.

Correct Answer. a

(148). Which of the following is true regarding cyanosis in trauma patient

a. Early sign of hypoxia

b. Late sign of hypoxia

c. Absence of cyanosis indicates adequate airway

d. Absence of cyanosis indicates adequate tissue perfusion

Solution. Ans-148: (d) Absence of cyanosis indicates adequate tissue perfusion Ref: Read the text below Sol:
Cyanosis is the bluish discoloration of the skin and mucus membranes resulting from an increased concentration of reduced
haemoglobin (> 4 g/dl), or of haemoglobin derivatives (such as methemoglobinemiaand sulfhemoglobinemia), in the small blood vessels
of those areas. Usually marked in the lips, nail beds, and malar eminences.
Cyanosis may be subdivided into central and peripheral.
In the central type, the arterial oxygen saturation is reduced or an abnormal haemoglobin derivative is present, and the mucus
membranes and skin are both affected.
Peripheral cyanosis is due to slowing of blood flow and abnormally great extraction of O2 from normally saturated arterial blood. It
results from vasoconstriction and diminished peripheral blood flow, such as occurs in cold exposure, shock, CCF and peripheral vascular
disease. When cardiac output is reduced, as occurs in a patient with trauma, cutaneous vasoconstriction occurs as a compensatory
mechanism so that blood is diverted from the skin to more vital areas such as the CNS and heart, and cyanosis of the extremities may
result even though the arterial blood is normally saturated. Absence of cyanosis in a trauma patient rules out shock or that there is
adequate tissue perfusion.
Cyanosis is not a reliable sign of hypoxia. Severely anemic patients may never develop cyanosis, even though they are extremely hypoxic
because of an inadequate hemoglobin concentration (in such patients deoxygenated hemoglobin is never more than 4 gms/dl which is
required to produce cyanosis).

Correct Answer. d

Copyright 2014 Delhi Academy of Medical Sciences, All Rights Reserved. 56/76
(149). Negative intrapleural pressure is due to

a. Uniform distribution of surfactant over alveoli

b. Negative intraalveolar pressure

c. Absorption of lymphatics from pleura

d. Presence of cartilagenous ring in upper airways

Solution. Ans-149: (c) Absorption of lymphatics from pleura Ref: Ganong's Review of Medical Physiology23rd ed pg 596 Sol:
Intrapleural pressure (also k/a pleural pressure or thoracic pressure or esophageal pressure) is the pressure of the fluid in the thin space
between the lung pleura and the chest wall pleura.
Continuous drainage of excess fluid into the lymphatic channels maintains a slight suction and therefore a slightly negative pressure or
subatmospheric pressure between the two layers of the pleura.
The second reason for negative pressure in the pleural space is the tendency of the lungs to recoil towards the inside away from the
chest wall and the tendency of the chest wall to recoil in the opposite direction i.e., towards the outside. (If the chest wall is opened, the
lungs collapse; and if the lungs lose their elasticity, the chest expands and becomes barrel shaped).
A negative intrapleural pressure is relative to the atmospheric pressure. If the atmospheric pressure is 760 mms of Hg, an intrapleural
pressure of -2 mms of Hg means the pressure in the pleural space is 758 mms of Hg.
The normal pleural pressure at the beginning of inspiration is -5 cms of H2O (or -2.5 mms of Hg) and becomes -7.5 cms of H2O (or -6
mms of Hg).
Strong inspiratory efforts reduce the intrapleural pressure to values as low as -30 mms of Hg, producing correspondingly greater
degrees of lung inflation.
The intrapleural pressure becomes positive during the valsalva manoeuvre i.e., forced expiration against a closed glottis.

Correct Answer. c

(150). The chief cells of the stomach secrete

a. Intrinsic factor

b. Hydrochloric acid

c. Pepsinogen

d. Gastrin

Solution. Ans-150: (c) Pepsinogen Ref: Read the text below Sol:
The chief cells of the stomach secrete pepsinogen, and the parietal cells of the stomach secrete hydrochloric acid and intrinsic factor.
Gastrin and CCK are secreted by specialized endocrine cells.

Correct Answer. c

(151). True statements about the ABO blood group include:

a. Blood group O is the most common type.

b. Blood group A is the least common type.

c. Patients with blood group B is regarded as universal recipients.

d. Anti-A and anti-B antibodies are found in patients with blood group AB

Solution. Ans-151: (a) Blood group O is the most common type. Ref: Read the text below Sol:
. Blood group O is the most common and blood group AB is the least common.
. Blood group O is regarded as universal donors and blood group AB as universal recipient.
. Anti-A and anti-B antibodies are found in patients with blood group O.
. Blood group AB contains no antibodies against any ABO group.

Correct Answer. a

Copyright 2014 Delhi Academy of Medical Sciences, All Rights Reserved. 57/76
(152). In the ECG:

a. P wave indicates arterial repolarisation.

b. QRS complex indicates ventricular depolarisation.

c. T wave indicates ventricular depolarisation.

d. Increased PR interval occurs in second degree heart block.

Solution. Ans-152: (b) QRS complex indicates ventricular depolarisation Ref: Read the text below Sol:
The P waves indicate arterial depolarisation, the QRS complex ventricular depolarisation. T wave represents ventricular repolarisation.
Increased PR interval indicates first degree heart bloc
The normal QRS complex is less than 0.12 seconds

Correct Answer. b

(153). The cold pressor response is initiated by stimulation of

a. Baroreceptors

b. Cardiopulmonary receptors

c. Hypothalamic receptors

d. Pain receptors

Solution. Ans-153: (d) Pain receptors Ref: Read the text below Sol:
The cold pressor response is initiated by the stimulation of pain receptors by exposing the surface of the skin to ice water.

Correct Answer. d

(154). Which one of the following signs is observed in a patient who has lost a significant amount of blood?

a. Respiratory acidosis

b. Dry skin

c. Polyuria

d. Low hematocrit

Solution. Ans-154: (d) Low hematocrit Ref: Read the text below Sol :
Loss of blood causes blood pressure to fall. The baroreceptor reflex response to the fall in blood pressure causes arteriolar resistance to
increase, further decreasing capillary perfusing pressure.
Because whole blood is lost, the concentration of circulating proteins remains normal and, therefore, the oncotic pressure remains the
same. The decreased capillary pressure and normal oncotic pressure result in the transfer of fluid from the interstitium to the vascular
bed, decreasing the hematocrit.
The increased arteriolar constriction lowers blood flow to the kidney causing urine formation to decrease.
Sympathetic stimulation causes peripheral constriction and produces sweating, resulting in the classic sign of hemorrhage: cold, damp
skin. The baroreceptor reflex increases heart rate.

Correct Answer. d

Copyright 2014 Delhi Academy of Medical Sciences, All Rights Reserved. 58/76
(155). The secretion of ACTH is correctly described in which of the following statements?

a. It shows circadian rhythm in humans

b. It is decreased during periods of stress

c. It is inhibited by aldosterone

d. It is stimulated by glucocorticoids

Solution. Ans-155: (a) It shows circadian rhythm in humans Ref: Read the text below Sol :
The secretion of ACTH occurs in several irregular bursts during the day; the peak occurs early in the morning prior to awakening and
thus is not due to the stress of arising.
This circadian rhythmmaximum secretion in early morning, minimum in the eveningis regulated by the hypothalamus through the
secretion of corticotropin-releasing hormone (CRH) into the hypothalamichypophyseal portal capillary system.
In addition to the basal rhythm, physical or mental stress will lead to increased ACTH secretion within minutes.
ACTH is also regulated as a result of feedback inhibition by the hormones whose synthesis it stimulates, such as glucocorticoids.
Aldosterone is a mineralocorticoid and is not controlled by ACTH. Epinephrine does not appear to have any effect on ACTH secretion.

Correct Answer. a

(156). Which one of the following statements is correct?

a. About 100 mg of iron is absorbed per day

b. Iron is absorbed rapidly from the small intestine

c. Iron is transported in the blood bound to transferrin

d. In general, iron must be oxidized from the ferrous to the ferric state for efficient absorption

Solution. Ans-156: (c) Iron is transported in the blood bound to transferrin Ref: Read the text below Sol :
Iron is transported in the blood bound to the beta globulin, transferrin. Excess iron is stored in all cells, but especially in hepatocytes
where it combines with apoferritin. The stored form is called ferritin.
The rate of iron absorption is extremely slow, with a maximum of only a few milligrams per day. Iron is absorbed primarily in the ferrous
form. Therefore, ferrous iron compounds, rather than ferric compounds, are effective in treating iron deficiency.

Correct Answer. c

(157). Inactivation of phospholamban in cardiac muscle decreases

a. The velocity of contraction in smooth muscle

b. The duration of contraction

c. The strength of contraction

d. The depolarization required to initiate a contraction

Solution. Ans-157: (b) The duration of contraction Ref: Read the text below Sol :
Phospholamban is a protein contained within the sarcoplasmic reticulum that inhibits the activity of the SR calcium pump.
Inactivation of phospholamban results in an increase in calcium sequestration by the SR. In cardiac muscle, the rapid sequestration of
calcium shortens the duration of the contraction.
In smooth muscle, calcium sequestration causes the muscle to relax. Phospholamban has little effect on skeletal muscle contraction.

Correct Answer. b

Copyright 2014 Delhi Academy of Medical Sciences, All Rights Reserved. 59/76
(158). Which of the following statements about synaptic transmission at the neuromuscular junction is true?

a. It is enhanced by high levels of cholinesterase

b. It is depressed by abnormally low levels of magnesium

c. It is depressed by increased parasympathetic nerve activity

d. It is produced by the release of acetylcholine from the alpha motoneuron

Solution. Ans-158: (d) It is produced by the release of acetylcholine from the alpha motoneuron Ref: Read the text below Sol :
Synaptic transmission at the myoneural junction is produced by the release of acetylcholine from the nerve terminal of the alpha
motoneuron.
High levels of cholinesterase would tend to interfere with synaptic transmission, as would high (not low) levels of magnesium.
The end-plate potential is produced by the flow of Na+ through channels permeable to both Na+ and K+.

Correct Answer. d

(159). Warfarin (coumarin) is an anticoagulant that is often given to patients following a heart attack. However, if too much warfarin is
administered, the patient can have episodes of bleeding. The bleeding produced by warfarin can be overcome by administering

a. Aspirin

b. Heparin

c. t-PA (tissue plasminogen activator)

d. Vitamin K

Solution. Ans-159: (d) Vitamin K Ref: Read the text below Sol :
Warfarin is often prescribed for patients at risk for thromboembolic episodes. Vitamin K is necessary for the conversion of prothrombin
to thrombin.
Thrombin is an important intermediate in the coagulation cascade. It converts fibrinogen to fibrin and is a powerful activator of
platelets.
Warfarin interferes with the activity of vitamin K and therefore reduces the likelihood of clot formation. Administering vitamin K can
restore coagulation if warfarin therapy leads to excessive bleeding.
Heparin prevents clotting by inhibiting thrombin. Aspirin blocks the formation of thromboxane A2, which is necessary for platelet
aggregation. Tissue plasminogen activator (t-PA) is a thrombolytic agent.

Correct Answer. d

(160). Negative-feedback control systems

a. Would not apply to the regulation of PaCO2

b. Give the best control when most sensitive

c. Are ineffective if the properties of the controlled system change

d. Are not necessarily stable

Solution. Ans-160: (d) Are not necessarily stable Ref: Read the text below Sol:
Negative-feedback systems are not necessarily the most stable.

Correct Answer. d

Copyright 2014 Delhi Academy of Medical Sciences, All Rights Reserved. 60/76
(161). With regard to the control of minute ventilation by carbon dioxide

a. About 80% of the effect of PaO2 is mediated by the peripheral chemoreceptors

b. Central effects are mediated by direct effects on cells of the DRG/VRG complex

c. Sensitivity of the control system is inversely related to the prevailing PaO2

d. This mechanism is less sensitive than control in response to oxygen

Solution. Ans-161: (c) Sensitivity of the control system is inversely related to the prevailing PaO2 Ref: Read the text below Sol:
The control of ventilation by PaCO2 works primarily through the central chemoreceptors.
However, the central effects are mediated indirectly through a change in CSF [H+], and the sensitivity is inversely related to PaO2.

Correct Answer. c

(162). In a kidney producing urine with an osmolality of 1,200 mOsm/kg H2O,the osmolality of fluid collected from the end of the cortical
collecting duct is about

a. 100 mOsm/kg H2O

b. 300 mOsm/kg H2O

c. 600 mOsm/kg H2O

d. 900 mOsm/kg H2O

Solution. Ans-162: (b) 300 mOsm/kg H2O Ref: Read the text below Sol:
When the kidney is producing maximally concentrated urine, fluid in the cortical collecting duct becomes isosmotic with the surrounding
cortical interstitial fluid.
Therefore, the osmolality will be about 300 mosm/kg H2O; it cannot go above this value because hyperosmotic values (compared to
systemic blood plasma) can be produced only in the kidney medulla

Correct Answer. b

(163). If the plasma concentration of a freely filterable substance is 2 mg/mL,GFR is 100 mL/min, urine concentration of the substance is 10
mg/mL, and urine flow rate is 5 mL/min, we can conclude that the kidney tubules

a. Reabsorbed 150 mg/min

b. Reabsorbed 200 mg/min

c. Secreted 50 mg/min

d. Secreted 150 mg/min

Solution. Ans-163: (a) Reabsorbed 150 mg/min Ref: Read the text below Sol:
The filtered load of the substance is Px GFR = 2 mg/mL 100 mL/min =200 mg/min.
The rate of excretion is Ux =10 mg/mL 5 mL/min = 50 mg/min.
Hence, more substance X was filtered than was excreted, and the difference, 200 mg/min-- 50 mg/min = 150 mg/min, gives the rate of
tubular reabsorption of substance X

Correct Answer. a

Copyright 2014 Delhi Academy of Medical Sciences, All Rights Reserved. 61/76
(164). An ECG showing two P waves preceding each QRS complex. Interpretation of this pattern is:-

a. Decreased conduction through AV node.

b. Increased conduction through Purkinje system

c. Decreased firing rate of SA node

d. Increased firing rate of SA nodes

Solution. Ans-164: (a) Decreased conduction through AV node. Ref: Read the text below Sol:
A pattern of two P waves preceeding a single QRS complex means every alternate P wave is conducted towards Ventricular system & a
conduction delay occurs at AV node.

Correct Answer. a

(165). Which autonomic receptor mediates secretion of epinephrine by adrenal medulla :-

a. Adrenergic 1 receptor

b. Adrenergic 2 receptor

c. Cholinergic nicotinic receptor

d. Cholinergic muscarinic receptor

Solution. Ans-165: (c) Cholinergic nicotinic receptor


Ref: Read the text below
Sol:
Preganglionic sympathetic fibres synapse on the chromaffin cells, of adrenal medulla at a nicotinic receptor.

Correct Answer. c

(166). The most prominent effect of hypoxic hypoxia is:-

a. Hyperpnea

b. Muscular incoordination

c. Headache

d. Lassitude

Solution. Ans-166: (d) Lassitude


Ref: Read the text below
Sol:
Hypoxic hypoxia leads to depression of all parts of central nervous system and more so of cerebral cortex as it
is more sensitive to low PO2.
This result in lassitude.

Correct Answer. d

Copyright 2014 Delhi Academy of Medical Sciences, All Rights Reserved. 62/76
(167). Following the administration of Thyroid-stimulating hormone (TSH), the earliest effect seen is :-

a. Increased thyroglobulin proteolysis

b. Increased thyroxine synthesis

c. Increased iodide trapping by thyroid

d. Increased size of acinar cells

Solution. Ans-167: (a) Increased thyroglobulin proteolysis


Ref: Read the text below
Sol:
Increased proteolysis of the stored thyroglobulin in the follicles is the earliest effect of TSH administration.
This results in the release of thyroid hormones into circulating blood.
Then follows the iodide trapping, iodination of tyrosine and coupling to form the thyroid hormones.

Correct Answer. a

(168). A man has progressive, chronic kidney disease. Which of the following indicates the greatest absolute decrease in GFR?

a. A fall in plasma creatinine from 4 mg/dL to 2 mg/dL

b. A fall in plasma creatinine from 2 mg/dL to 1 mg/dL

c. A rise in plasma creatinine from 1 mg/dL to 2 mg/dL

d. A rise in plasma creatinine from 2 mg/dL to 4 mg/dL

Solution. Ans-168: (c) A rise in plasma creatinine from 1 mg/dL to 2 mg/dL


Ref: Read the text below
Sol:
There is an inverse hyperbolic relationship between plasma [creatinine] and GFR and, therefore, a rise in
plasma [creatinine] is associated with a fall in GFR (see Fig. 23.7).
The greatest absolute change in GFR occurs when plasma [creatinine] doubles starting from a normal GFR and
plasma [creatinine].

Correct Answer. c

Copyright 2014 Delhi Academy of Medical Sciences, All Rights Reserved. 63/76
(169). Which of the following body fluid volumes cannot be directly determined with a single indicator?

a. Extracellular fluid volume

b. Intracellular fluid volume

c. Plasma volume

d. Total body water

Solution. Ans-169: (b) Intracellular fluid volume


Ref: Read the text below
Sol:
ICF volume is calculated by subtracting ECF volume from the total body water.
The other fluid volumes can be determined from the volume of distribution of a single indicator, such as
radioactive sulfate for ECF volume, radioiodinated serum albumin for plasma volume, and deuterium oxide for
total body water.

Correct Answer. b

(170). Regarding the cerebrospinal fluid choose incorrect statement:

a. The glucose level is lower than that of venous blood.

b. The pH is lower than that of arterial blood.

c. The protein level is lower than that of arterial blood.

d. The PO2 is the same as that of arterial blood.

Solution. Ans-170: (d) The PO2 is the same as that of arterial blood Ref: Read the text below Sol:
The pH and the PO2 of the CSF are lower than that of the arterial blood.
Protein is almost absent in the normal CSF.
The glucose in CSF is 2/3 that of the blood. The chloride level in CSF is higher than that of the arterial blood.

Correct Answer. d

(171). Caisson's disease is due to

a. Gas embolism

b. Fat embolism

c. Amniotic fluid

d. Tumor embolism

Solution. Ans-171: (a) Gas embolism Ref: Ref: Guyton, 11th edition pg 548-549 Sol:
Caissons disease (synonyms- Decompression sickness, bends, Compressed air sickness, Divers paralysis, Dysbarism)- symptoms of
caissons disease are caused by gas bubbles blocking many blood vessels in different tissues (gas embolism) seen during sudden
decompression in divers and even pilots.
If a diver has been beneath the sea long enough that large amounts of nitrogen have dissolved in his or her body and the diver then
suddenly comes back to the surface of the sea, significant quantities of nitrogen bubbles can develop in the body fluids either
intracellularly or extracellularly and can cause minor or serious damage in almost any area of the body, depending on the number and
sizes of bubbles formed; this is called decompression sickness
Most common symptom of decompression sickness, affecting 85 to 90% of patients is bends or pain in the joints and muscles of legs
and arms. In 5 to 10% of the patient nervous system symptoms occur ranging from dizziness to paralysis or collapse and
unconsciousness. 2% of patients develop the chokes caused by gas embolism in the capillaries of the lungs; patient complains of
dyspnoea followed by severe pulmonary edema and occasionally, death.

Correct Answer. a

Copyright 2014 Delhi Academy of Medical Sciences, All Rights Reserved. 64/76
(172). All are features of smooth muscle except:

a. Plasticity

b. Troponins

c. Latch bridge mechanism

d. L tubules

Solution. Ans-172: (b) Troponins Ref.: Read the text below Sol : Smooth muscles Structural Differences from a Skeletal Muscle
No cross-striations
Smooth muscles contain relatively less thick filaments and more thin filaments
Instead of Z lines, there are dense bodies in the cytoplasm
Instead of troponin, a calcium binding protein calmodulin is present.
Tropomyosin not covers the active site of action
Phosphorylation of myosin is necessary for the contraction of the smooth muscle
Poorly developed sarcoplasmic reticulum
T-tubules are absent
L-tubules are present but the do not form terminal cisterns
Few mitochondria & depends mostly on glycolysis for their metabolic needs
No length-tension relationship exist because of its plasticity (Tension does not increase after a particular point of stretch

Correct Answer. b

(173). EEG waves are also known as Bergers rhythm. EEG wave with highest amplitude are seen during which of the following states?

a. Drowsy

b. Agitated

c. Alert

d. Deep sleep

Solution. Ans-173: (d) Deep sleep


Electro Encephalo Gram

Correct Answer. d

Copyright 2014 Delhi Academy of Medical Sciences, All Rights Reserved. 65/76
(174). True about REM sleep is:

a. Occupies 80% of total sleep time in the elderly

b. First state of sleep when a person falls asleep

c. Reduced muscle tone throughout the body

d. Associated with night terror

Solution. Ans-174: (c) Reduced muscle tone throughout the body Ref.: Read the text below Sol : Rem sleep (Rapid eye movement)
20-25% of sleep
REM sleep lasting 5-30 minutes usually appear on the average every 90 minutes of NREM sleep, in a normal night sleep. It is more
towards morning & 4-6 REM periods per night
REM sleep occupies about
80% of total sleep time in premature infants
50% in full term neonates
25% in young adults
20% in the elderly

Correct Answer. c

(175). True about muscle spindles are all except:

a. Gamma motor neurons supply the intrafusal fibres

b. Intrafusal fibres serve a pure sensory function

c. Intrafusal fibres contain contractile centre & non-contractile ends

d. Muscle spindle detects changes in muscle length

Solution. Ans-175: (c) Intrafusal fibres contain contractile centre & non-contractile ends Ref.: Read the text below Sol : Each spindle is
surrounded by two types of fibres.
Situated inside the spindle : intrafusal fibres (-12 in number) &
Situated outside the spindle : extrafusal fibres (regular contractile units of the muscle supplied by motor neurons)
Intrafusal fibres contain non-contractile centre (no actin & myosin) and contractile polar ends
Intrafusal fibres serve a pure sensory function
Two types of intrafusal fibres :

Correct Answer. c

Copyright 2014 Delhi Academy of Medical Sciences, All Rights Reserved. 66/76
(176). True about cerebellum are all except:

a. Purkinje cell axons are the only output from cerebellar cortex

b. Medial most nucleus of the cerebellum is fastigial nucleus

c. Output of the deep cerbellar nuclei is always excitatory

d. Parallel fibres are formed by Purkinje cells

Solution. Ans-176: (d) Parallel fibres are formed by Purkinje cells Ref.: Read the text below Sol :
Purkinje cells are present in the molecular layer. Their axons are the only output from the cerebellar cortex and ends in deep cerebellar
nuclei which is always inhibitory.
Climbing fibres stimulate purkinje cells directly whereas mossy fibres stimulate purkinje cells via parallel fibres.
Granule cell present in the granular layer receives excitatory input from the mossy fibres.
Their axons extend through the molecular layer & form parallel fibres which gives excitatory input to purkinje cells, basket cells, stellate
cells and golgi cells.

Correct Answer. d

(177). The Rh antigens are found in:

a. All nucleated cells

b. WBCs

c. Only RBCs

d. All blood cells

Solution. Ans-177: (c) Only RBCs Ref.: Read the text below Sol : ABO System
The genes for ABO system is located on chromosome 9
Four blood types on the basis of A and B antigens on the cell membrane of RBCs inherited as mendeliaan dominants.
Type A individuals have the A antigen, type B individuals have the B, type AB have both, and type O have neither.
An H gene codes for a fucose transferase that adds a terminal fucose, forming the H antigen that is usally present in all individuals of all
blood types.
Antibodies against red cell agglutinogens are called agglutinins

Correct Answer. c

(178). Which phase of cardiac cycle is best suited for cardiac imaging?

a. Diastasis

b. Isovolumetric contraction

c. Isovolumetric relaxation

d. Midsystole

Solution. Ans-178: (a) Diastasis Ref.: Read the text below Sol :
Phase of minimum cardiac motion : Diastasis (mid-diastole) at normal HR & Late systole (at high heart rates)

Correct Answer. a

Copyright 2014 Delhi Academy of Medical Sciences, All Rights Reserved. 67/76
(179). Central venous pressure is increased by:

a. Increased venous compliance

b. Decreased blood volume

c. Decreased heart rate

d. Increased peripheral resistance

Solution. Ans-179: (c) Decreased heart rate Ref.: Read the text below Sol : Central venous pressure (CVP)
Central venous pressure describes the pressure in the thoracic vena cava near the Rt. Atrium.
Therefore; CVP and Rt. Atrial pressure are essentially the same. It is measured in Right Internal Jugular Vein.
The CVP averages 2-6 mmHg or 5-10 cm H2O but fluctuates with respiration and heart action.
CVP=V/CV where, V = change in the volume of blood within the thoracic veins, CV= compliance of veins

Correct Answer. c

(180). Waterfall effect is seen in which zone of the lung?

a. Zone 1

b. Zone 2

c. Zone 3

d. Zone 4

Solution. Ans-180: (b) Zone 2 Ref.: Read the text below Sol :
Waterfall effect is seen in zone 2 of the lung

Correct Answer. b

Copyright 2014 Delhi Academy of Medical Sciences, All Rights Reserved. 68/76
(181). Which of the following is not absorbed in PCT

a. Na+

b. HCO3

c. PO4

d. H+

Solution. Ans-181: (d) H+ Ref: Ganong's Review of Medical Physiology23rd ed pg 648 Sol:

Correct Answer. d

(182). Which of the following occurs after 24 hours of fasting:

a. Lipolysis

b. Muscle break-down.

c. Hepatic gluconeogenesis.

d. All of above.

Solution. Ans-182: (d) All of above Ref: Read the text below Sol:
After 24 hours of fasting, the liver undergoes gluconeogenesis as the storage of glycogen is usually insufficient to maintain the blood
glucose level.
Lipolysis also occurs.
Muscle break-down does not occur until much later. The blood glucose concentration is usually maintained.
The brain switches from using glucose to ketone bodies which are derived from fatty acid oxidation in the level.

Correct Answer. d

Copyright 2014 Delhi Academy of Medical Sciences, All Rights Reserved. 69/76
(183). The electrically neutral active transport of sodium from the lumen of the kidney occurs in the

a. Proximal tubule

b. Descending limb of the loop of Henle

c. Ascending limb of the loop of Henle

d. Cortical collecting duct

Solution. Ans-183: (c) Ascending limb of the loop of Henle Ref: Read the text below Sol :
In the ascending limb of the loop of Henle, sodium is actively transported by a transporter that has one binding site for sodium, one
binding site for potassium, and two binding sites for chloride.
Thus, the transport of sodium is electrically neutral. Electrically neutral sodium transport also occurs in the distal convoluted tubule and
connecting segment by a carrier that binds both sodium and chloride.
In all other segments of the nephron, however, sodium transport is electrogenic; that is, sodium is transported unaccompanied by a
negatively charged chloride ion.

Correct Answer. c

(184). Which one of the following characteristics is most similar in the systemic and pulmonary circulations?

a. Stroke work

b. Preload

c. Afterload

d. Peak systolic pressure

Solution. Ans-184: (b) Preload Ref: Read the text below Sol :
The right and left ventricles are in series with one another so that the entire cardiac output (except for a small anatomic shunt) passes
through both circulations.
Since the two ventricles beat at the same rate, their stroke volumes are the same.
However, the resistance of the pulmonary vasculature is much smaller than that of the systemic circulation; thus, the afterload and
stroke work are greater on the left side than on the right side.
Because the same cardiac output is ejected into a higher resistance, peak systolic pressure is higher on the left side than on the right
side.
Only about 10% of the blood volume is within the pulmonary circulation at any one time. About two-thirds of the blood volume is stored
within the systemic veins and venules. Although the left and right preloads are not identical, they are very similar.

Correct Answer. b

(185). The effect of insulin on glucose transport is to

a. Permit transport against a concentration gradient

b. Enhance transport into adipocytes

c. Enhance transport across the tubular epithelium of the kidney

d. Enhance transport into the brain

Solution. Ans-185: (b) Enhance transport into adipocytes Ref: Read the text below Sol :
Insulin increases glucose uptake by adipocytes.
Transport of glucose into cells is by facilitated diffusion. Insulin increases the number of transporters available for glucose uptake in
many cells, including adipocytes, skeletal and cardiac muscle, and some smooth muscle.
Insulin does not enhance glucose transport into brain cells, intestinal mucosal cells, or renal tubular epithelial cells.

Correct Answer. b

Copyright 2014 Delhi Academy of Medical Sciences, All Rights Reserved. 70/76
(186). An increase in sodium conductance is associated with

a. The downstroke of the skeletal muscle action potential

b. The upstroke of the smooth muscle action potential

c. The refractory period of the nerve cell action potential

d. The end-plate potential of the skeletal muscle fiber

Solution. Ans-186: (d) The end-plate potential of the skeletal muscle fiber Ref: Read the text below Sol :
The channel opened by Ach when it binds to receptors on the end plates of skeletal muscle fibers is equally permeable to potassium and
sodium.
The increase in sodium permeability allows sodium to flow into the cell and produces the end-plate potential.
The plateau phase of ventricular muscle action potentials and the upstroke of smooth muscle action potentials are produced by an
increase in calcium conductance. An increase in potassium conductance is responsible for the downstroke of the action potential.
The refractory period is caused by an increase in potassium conductance and a decrease in the number of sodium channels available to
produce an action potential (i.e., sodium channel inactivation).

Correct Answer. d

(187). True about myocardial O2 demand is

a. Directly proportional to duration of systole

b. Inversely proportional to heart rate

c. Negligible in quiescent heart

d. Has a constant relation to the external work done by heart

Solution. Ans-187: (a) Directly proportional to duration of systole Ref: Ganong 22nd,pg 275 Sol : Maximum oxygen consumption at rest
per 100 gms of tissue is by the heartQ (9.7 mL/100 gm of tissue/min; the values for brain and liver are 3.3 and 2.0 mL/100 gm/min). Since
at rest the heart extracts 70-80% of O2 from each unit of blood delivered to it, an increase in O2 demand can therefore be met, not by
increasing oxygen extraction, but by increasing the coronary blood flow. Since the most important determinant of myocardial O2
consumption is the mechanical activity of the heart and its inotropic state, the determinants of myocardial O2 consumption are also the
determinants of coronary blood flow. They include o Heart rate (there is a linear relationship between increase in heart rate and increase
in myocardial O2 demand- more the heart rate, more the myocardial O2 demand and more the coronary blood flow) o Systolic ventricular
pressure or wall tension (an almost linear increase in myocardial demand results as the peak systolic pressure or the peak systolic
tension developed by the left ventricle is increased)
Amount of shortening against a load- shortening of the muscle in the ventricular wall as the stroke volume is ejected i.e., the duration of
systole, is a less important but a significant determinant of myocardial O2 consumption
Work done by the heart (work done = stroke volume X mean pressure; increase in volume work is less costly in terms of O2 consumption
than the same amount of work performed against a high pressure)
Level of inotropic state or contractility- when a positive inotropic substance such as calcium, digitalis or nor epinephrine is administered
an additional increase in myocardial O2 consumption is seen The close relationship between coronary blood flow and myocardial O2
consumption indicates that one or more of the products of metabolism causecoronary vasodilation.

Correct Answer. a

(188). Systemic arteriolar constriction may result from an increase in the local concentration of

a. Nitric oxide

b. Angiotensin II

c. Atrial natriuretic peptide

d. Beta () agonists

Solution. Ans-188: (b) Angiotensin II Ref: Read the text below Sol :
Angiotensin II is a powerful vasoconstrictor that is formed when renin is released from the kidney in response to a fall in blood pressure
or vascular volume.
Renin converts angiotensinogen to angiotensin I.
Angiotensin II is formed from angiotensin I by an angiotensin-converting enzyme localized within the vasculature of the lung. All the
other listed substances cause vasodilation.

Correct Answer. b

Copyright 2014 Delhi Academy of Medical Sciences, All Rights Reserved. 71/76
(189). Which of the following fluids offers the greatest expansion of the extracellular fluid ?

a. Albumin

b. 5% dextrose in water

c. Normal saline

d. Packed red blood cells

Solution. Ans-189: (d) Packed red blood cells Ref: Read the text below Sol :
Packed red blood cells stay entirely in the vascular bed. Other fluids will distribute between the intracellular and extracellular fluid
compartments.
The division of the distribution will depend on the amount of solute

Correct Answer. d

(190). One liter of normal saline will expand the extracellular fluid volume by

a. 1%

b. 2%

c. 6%

d. 50%

Solution. Ans-190: (c) 6% Ref: Read the text below Sol :


Of a liter of normal saline, about 300 mL will remain in the vascular bed, representing about a 6% increase in volume.

Correct Answer. c

(191). Which of the following characteristics of an axon is most dependent on its diameter?

a. The magnitude of its resting potential

b. The duration of its refractory period

c. The conduction velocity of its action potential

d. The overshoot of its action potential

Solution. Ans-191: (c) The conduction velocity of its action potential Ref: Read the text below Sol :
The conduction velocity of an action potential along an axon is proportional to the axons diameter for both unmyelinated and myelinated
axons.
In myelinated axons, the conduction velocity of an action potential also increases as the distance between nodes of Ranvier increases.
The resting potential and the action potential of a nerve axon are dependent on the type and density of electrically excitable gates and
the ability of the Na+,K+-ATPase to establish and maintain the concentration gradients.
These characteristics are not related in any systematic way to the axon diameter.

Correct Answer. c

Copyright 2014 Delhi Academy of Medical Sciences, All Rights Reserved. 72/76
(192). Prolactin secretion increased by all except:

a. Dopamine

b. Sleep

c. Pregnancy

d. Stress

Solution. Ans-192: (a) Dopamine Ref.: Read the text below Sol :
Dopamine is the primary neuroendocrine inhibitor of the secretion of prolactin from the anterior pituitary gland and hence causes
hypoprolactinemia not hyperprolacinnemia

Correct Answer. a

(193). Renshaw cells are inhibitory interneurons found in?

a. Gray matter of brain

b. White matter of brain

c. Gray matter of the spinal cord

d. White matter of spinal cord

Solution. Ans-193: (c) Gray matter of the spinal cord Ref.: Read the text below Sol :
Renshaw cells of grey matter of spinal cord produces self-inhibition of motor neurons.
The axons of motor neurons send branches that make excitatory synaptic connection onto Renshaw cells.
The Renshaw cells in turn make inhibitory synaptic connections back onto the same motor neuron.

Correct Answer. c

(194). True about volume receptors is all except:

a. They are low pressure receptors

b. They provide afferents for thirst control

c. They are located in carotid sinus

d. They mediate vasopressin release

Solution. Ans-194: (c) They are located in carotid sinus Ref.: Read the text below Sol :

Correct Answer. c

Copyright 2014 Delhi Academy of Medical Sciences, All Rights Reserved. 73/76
(195). Normal nasal potential difference is?

a. More than-10mv

b. More than 20 mv

c. More than 30 mv

d. More than -40 mv

Solution. Ans-195: (d) More than -40 mv Ref.: Read the text below Sol :
Nasal potential difference test is done for the diagnosis of cystic fibrosis.
It is not used routinely for screening test because it is invasive.
The test involves one electrode that is placed on the nasal mucosa of the inferior turbinate, and another subcutaneously on the forearm,
connected to a voltmeter to measure the potential difference. A nasal potential difference less than 40mv is considered abnormal.

Correct Answer. d

(196). Chronaxie minimum in?

a. Mixed nerves

b. Unmyelinated nerve

c. Myelinated nerve

d. Sensory nerves

Solution. Ans-196: (c) Myelinated nerve Ref.: Read the text below Sol :
Chronaxie (or chronaxy) is the minimum time over which an electric current, double the strength of the rheobase, needs to be applied, in
order to stimulate a muscle fiber or nerve cell.
Rheobase is the minimal current amplitude of indefinite duration (practically, a few hundred milliseconds) that result in the
depolarization threshold of the cell membranes being reached (i.e. an action potential or the contraction of a muscle).
In the case of a nerve or single muscle cell, rheobase is half the current that needs to be applied for the duration of chronaxie to result
in an action potential or muscle twitch.
Remember chronaxie is the measure of excitability of the nerve. Hence chronaxie is inversely proportional to the myelination of the
nerves.
Order of neurons in order of increasing chronaxie : A alpha > A delta > C

Correct Answer. c

(197). Transducin is a protein found in?

a. Glomerulus

b. Retina

c. Skeletal muscle

d. Adrenal medulla

Solution. Ans-197: (b) Retina Ref.: Read the text below Sol :
Transducin is a heterotrimeric G protein that is naturally expressed in vertebrate retina rods and cones

Correct Answer. b

Copyright 2014 Delhi Academy of Medical Sciences, All Rights Reserved. 74/76
(198). Which of the following is a Relaxation protein?

a. Actin

b. Myosin

c. Tropomyosin

d. Dystrophin

Solution. Ans-198: (c) Tropomyosin Ref.: Read the text below Sol :
The troponin-tropomyosin complex so called relaxing proteins which inhibit the interaction between actin and myosin.

Correct Answer. c

(199). SHBG is decreased in?

a. Hyperthyroidism

b. Increased androgen

c. Increased estrogen

d. Pregnancy

Solution. Ans-199: (b) Increased androge Ref.: Read the text below Sol :

Correct Answer. b

(200). Hypothyroidism causes CNS features because of presence of which receptor in brain?

a. TR alpha 1

b. TR alpha 2

c. TR beta 1

d. TR beta 2

Solution. Ans-200: (d) TR beta 2 Ref.: Read the text below Sol :
There are two human TR genes: an receptor gene on chromosome 17 and a receptor gene on chromosome 3.
By alternative splicing, each forms at least two different mRNAs and therefore two different receptor proteins.
TR2 is found only in the brain, but TR1, TR2, and TR1 are widely distributed.

Correct Answer. d

Test Answer
1.(d) 2.(a) 3.(c) 4.(a) 5.(d) 6.(d) 7.(d) 8.(c) 9.(d) 10.(c)

11.(b) 12.(d) 13.(a) 14.(b) 15.(d) 16.(a) 17.(a) 18.(d) 19.(b) 20.(a)

21.(b) 22.(c) 23.(b) 24.(b) 25.(c) 26.(c) 27.(c) 28.(a) 29.(d) 30.(c)

31.(c) 32.(a) 33.(d) 34.(d) 35.(a) 36.(d) 37.(b) 38.(d) 39.(b) 40.(d)

41.(a) 42.(c) 43.(b) 44.(b) 45.(d) 46.(c) 47.(d) 48.(d) 49.(c) 50.(c)

Copyright 2014 Delhi Academy of Medical Sciences, All Rights Reserved. 75/76
51.(d) 52.(c) 53.(b) 54.(b) 55.(c) 56.(b) 57.(b) 58.(d) 59.(b) 60.(c)

61.(a) 62.(a) 63.(b) 64.(d) 65.(c) 66.(c) 67.(b) 68.(c) 69.(b) 70.(c)

71.(b) 72.(c) 73.(b) 74.(a) 75.(a) 76.(b) 77.(b) 78.(a) 79.(b) 80.(c)

81.(d) 82.(b) 83.(b) 84.(c) 85.(d) 86.(b) 87.(d) 88.(c) 89.(c) 90.(d)

91.(c) 92.(a) 93.(c) 94.(b) 95.(c) 96.(a) 97.(a) 98.(a) 99.(d) 100.(b)

101.(b) 102.(a) 103.(b) 104.(c) 105.(d) 106.(c) 107.(d) 108.(b) 109.(a) 110.(c)

111.(c) 112.(b) 113.(d) 114.(b) 115.(a) 116.(c) 117.(a) 118.(a) 119.(d) 120.(b)

121.(b) 122.(c) 123.(d) 124.(d) 125.(c) 126.(b) 127.(b) 128.(c) 129.(b) 130.(a)

131.(b) 132.(b) 133.(c) 134.(d) 135.(b) 136.(c) 137.(c) 138.(b) 139.(b) 140.(a)

141.(a) 142.(a) 143.(b) 144.(b) 145.(b) 146.(d) 147.(a) 148.(d) 149.(c) 150.(c)

151.(a) 152.(b) 153.(d) 154.(d) 155.(a) 156.(c) 157.(b) 158.(d) 159.(d) 160.(d)

161.(c) 162.(b) 163.(a) 164.(a) 165.(c) 166.(d) 167.(a) 168.(c) 169.(b) 170.(d)

171.(a) 172.(b) 173.(d) 174.(c) 175.(c) 176.(d) 177.(c) 178.(a) 179.(c) 180.(b)

181.(d) 182.(d) 183.(c) 184.(b) 185.(b) 186.(d) 187.(a) 188.(b) 189.(d) 190.(c)

191.(c) 192.(a) 193.(c) 194.(c) 195.(d) 196.(c) 197.(b) 198.(c) 199.(b) 200.(d)

Copyright 2014 Delhi Academy of Medical Sciences, All Rights Reserved. 76/76

Anda mungkin juga menyukai